SlideShare una empresa de Scribd logo
1 de 56
Enam jam selepas dipasangkan pengaliarn under water seal, Puan Rama mengadu
kesukaran bernafas. Tindakan segera anda ialah
a. Lakukan sedutan nasofaringeal
b. Nasihatkan pesakit menarik nafas panjang
c. Mulakan terapi oksigen 4 liter seminit
d. Periksa fluktasi paras air pengaliran under water seal
2. Intervensi kejururawatan yang dilakukan ke atas pesakit emphysema untuk
mengelakkan small airway collapse adalah
a. Melakukan postural drainage
b. Melakukan chest percussion and vibration
c. Galakkan pesakit melakukan pursed lip breathing
d. Galakkan pesakit melakukan deep breathing exercise
3. Nasihat yang perlu diberikan kepada pesakit yang menghidap bronchitis bagi
mencegah simptomnya menjadi lebih teruk adalah
a. Lakukan senaman awal pagi
b. Makan makanan yang tinggi fiber
c. Elakkan dari tempat sesak dan berhabuk
d. Hentikan antibiotic hanya apabila keadaan pulih
4. Cik Lim dipasang chest tube sebab mengalami haemotorax. Tujuan merawat
beliau dalam posisi semi fowlers ialah untuk
a. Mengelakkan pulmonary emboli
b. Memastikan tiub penyaliran dada berfungsi dengan baik
c. Menggalakkan penyaliran dan perkembangan paru-paru
d. Membenarkan jururawat mencapai tiub dada dengan mudah
5. Chest tube dimasukkan pada Puan Salmi kerana mengalami pneumotorax.
Chest tube diklamkan sebelum pengeluaran. Tiba-tiba beliau mengalami
dispnea. Intervensi kejururawatan segera adalah
a. Membuka klam
b. Beri terapi oksigen
c. Posisi semula pesakit
d. Ajar pesakit senaman pernafasan
6. Encik Ravi telah dijadualkanuntuk menjalani torakotomi pada keesokkan
hari. Tindakan kejururawatan untuk mengekalkan perkembangan paru-paru
adalah
a. Mengawal pengambilan cecair
b. Mengekalkan rehat sepenuhnya
c. Meletakkan klien pada posisi fowlers
d. Mengajar klien senaman pernafasan
7. Encik Maniam mengalami chronic obstructive pulmonary disease. Intervensi
kejururawatan untuk meningkatkan status nutrisi beliau adalah
a. Mulakan hyperalimentation therapy
b. Beri diet kalori tinggi mengikiut toleransi
c. Meningkatkan paras oksigen sebelum makan
d. Beri sedikit makanan berzat dengan kerap
8. Seorang klien dengan asthma mengalami wheezing. Jururawat memahami
ini berlaku kerana
a. Kurang pertukaran gas
b. Mucus yang pekat dan likat
c. Pengyekatan salur pernafasan
d. Penyekatan aliran udara yang keluar dari paru-paru
9. Anda diarahkan memberi infuse Amilophyline 1gm dalam 500ml Dextrose 5%
dalam tempoh masa 12 jam. Faktor titisan ialah 20. Berapakah kadar aliran
infuse seminit yang perlu anda berikan?
a. 5
b. 14
c. 83
d. 125
10. Tindakan bagi mengatasi masalah kejururawatan potensi kekurangan cecair
berkaitan dengan hemoraj bagi klien menghidap dengan haemorragic fever
ialah
a. Berikan nutrisi yang sesuai mengikut jumlah kehilangan cecair
b. Rawat klien dalam persekitaran yang selamat untuk mengelakkan
kecederaan
c. Beri cecair melalui tiub nasogastrik untuk menggantikan cecair yang hilang
d. Lakukan gastric lavage dengan air sejuk untuk memberhentikan pendarahan
11. Puan Lim mendesak untuk pulang walaupun masih belum discaj oleh doctor.
Apakah tindakan jururawat dalam situasi ini?
a. Puan Lim perlu menandatangani boring AOR sebab beliau ingkar arahan
doctor
b. Jangan hiiiraukan permintaan Puan Lim dan haling beliau dddaripada
meninggalkan hospital
c. Terangkan kepada Puan Lim bahawa beliau dibenarkan pulang sebaik sahaja
sembuh
d. Beritahu Puan Lim bahawa beliau tidak akan diterima masuk semula ke
hospital kerana ingkar arahan doctor
12. Encik Halim berumur 60 tahun mengalami cirit-birit yang teruk. Keputusan
gas darah arteri (ABG) beliau adalah pH 7.25, HCO3 18mEq/L, PCO2 32mmHg,
PO2 90mmHG dan BE -3. Keputusan beliau menunjukkan
a. Alkalosis metabolic
b. Alkalosis respiratori
c. Asidosis metabolic
d. Asidosis respiratori
13. Rosmah berumur 5 tahun diberi prednisolone untuk rawatan
glomerulonefritis kronik. Dalam memantau kesan sampingan medikasi ini,
jururawat perlu perhatikan untuk
a. Anoreksia dan sembelit
b. Tiada selera makan dan cirit birit
c. Tanda dehidrasi dan susut berat badan
d. Tanda retensi cecair dan penambahan berat badan
14. Cecair gentian pilihan untuk klien dewasa yang dimasukkan ke wad dengan
renjatan hipovolumia adalah
a. Mannitol
b. Dextrose 10%
c. Lactated Ringers
d. Normal saline 0.45%
15. Yang manakah antara berikut intervensi kejururawatan yang diambil jika
jumlah baki penyaliran dialysis peritoneum +1000ml?
a. Lapor segera kepada doctor
b. Panjangkan masa penyaliran keluar
c. Ubah baringan pesakit dari sisi ke sisi
d. Cepatkan kadar pengaliran masuk diasilat
16. Penyiasatan berikut adalah petunjuk yang unggul untuk menilai pengawalan
diabetes seorang pesakit datang ke klinik susulan selepas 3 bulan?
a. Fasting blood sugar
b. Random blood sugar
c. Analysis of microalbumin
d. Glycosylated haemoglobin level
17. Linda berumur 10 tahun seorang penghidap Type 1 Diabetes merancang
untuk mengikut kelas renang. Pendidikan pesakit yang perlu anda berikan
ialah
a. Tambahkan 10 unit regular insulin pada hari latihan
b. Tangguhkan masa makan malam sehingga selepas kelas renang
c. Mengawalatur dos insulin pada waktu pagi supaya peak action insulin
mencapai pada masa renang
d. Monitor paras glukos sebelum dan selepas berenang untuk mengubahsuai
amaun makanan dan insulin yang diperlukan
18. Berikut adalah kenyataan yang benar dalam pendidikan pesakit kepada Encik
Wong berusia 75 tahun dan kini menghidap Syndrome Inappropriate Anti
Dieuretic Hormone (SIADH)
a. Elakkan makan pisang dan oren
b. Kurangkan pengambilan garam daLam diet
c. Timbang dan rekod berat badan setiap hari
d. Minum sekurang-kurangnya 1 ½ hingga 2 liter cecair sehari
19. Pemerhatian yang penting terhadap seorang pesakit 4 jam selepas
tiroidektomi ialah
a. Corak pernafasan
b. Keserakan suara
c. Tingling of hands
d. Output urine
20. Puan Tina kembali ke wad selepas menjalani tiroidectomi dan pemerhatian
dijalankan setiap jam. Anda akan segera memanggil doctor sekiranya Puan
Tina mengalami
a. Muntah dan cirit birit
b. Kurang upaya menumpukan perhatian
c. Tachycardia dan peningkatan tekanan darah
d. Laryngeal stridor semasa inspirasi dan ekspirasi
A: SOALAN OBJEKTIF
Jawab semua soalan
1. Manakah tanda berikut menunjukkan angin ahmar?
A. Sakit facial
B. Pergerakan yang tidak stabil
C. Pertukaran bau
D. Hilang fungsi percakapan
2. Manakah berikut adalah manifestasi penyerangan iskemia transient?
A .Mula dengfan cepat dan symptom meningkat dengan perlahan
B. Symptom bermula dengan tiba-tiba dan dalam masa yang singkat
C. Symptom bermula dengan cepat dan menyebabkan kehilangan sedar diri
D. Symptom bermula dengan segera dan mengakibatkan kehilangan neurological.
3. Manakah intervensi berikut adalah kritikal dalam penjagaan klien selepas
lumbar punktur?
A. Puasakan klieMasukan kateter ‘indwelling fole
B.Beri sedatif pos prosedur untuk menggalakan tidur
C.Asses tempat punktur untuk pengeluaran cecair serebral spina artau
pembentukan hematoma
4. Manakah ubat berikut diberi untuk mencegah kejadian angin ahmar?
A Clonidine (Catapres)
B Benazepril (Lotensin)
C Methyldopa (Aldomet)
D Dipyridamole (persantin)
5. Nisbah compression : breathing dalam one-rescuer cardiopulmonary
resuscitation (CPR) ialah
A 5 : 1
B 5 : 2
C15 : 1
D15 : 2
6. Jumlah darah yang dipam keluar oleh jantung dalam satu denyutan jantung
dikenali sebagai
Preload
Afterload
Stroke volume
Cardiac output
7. Aritmia ialah satu keadaan gangguan kepada
Isipadu strok
Keluaran output
System konduksi jantung
Pengaliran darah ke arteri koronari
8. Hipertensi boleh mengakibatkan kegagalan jantung. Bahagian jantung yang
manakah akan mengalami kegagalan dahulu?
Atrium kiri
Atrium kanan
Ventrikel kiri
Ventrikel kanan
9. Pesakit hipertensi dinasihat mengurangkan pengambilan garam dalam
pemakanannya kerana garam akan
Menambahkan cecair intraselular
Menambahkan pengambilan vena
Meningkatkan retensi cecair interstitial
Menambahkan isipadu cecair intravascular
10.Pemerhatian yang perlu dilakukan ke atas pesakit yang mengalami cerebral
vascular accident (CVA) adalah tanda
A. Kegagalan ginjal
B. Kongesi pulmonary
C. Gangguan peredaran periferi
D. Peningkatan tekanan intrakranium
11.Encik Samad dimasukkan ke wad dengan tekanan darah 170/110 mmHg.
Tindakan kejururawatan pertama anda ialah
A. Rehatkan pesalit di atas katil
B. Lengkapkan data peribadi pesakit
C. Mengambil sejarah perubatan
D. Ambil tekanan darah serta merta
12.Mekanisma yang menyebabkan kesakitan dada dalam keadaan angina pectoris
ialah
A. tekanan darah yang tinggi
B. Kesesakan di arteri pulmonary
C. mengambil sejarah perubatan
D. ambil tekanan darah serta merta
13.Encik John yang mengalami infarksi miokardium dua hari lepas, kini didapati
demam low grade. Demam ini disebabkan oleh reaksi terhadap
A. Nekrosis tisu
B. Infeksi miokardium
C. Infeksi nasokomial
D. Bacterial endokarditis
14.Semasa pemberian ubat glyceryl trinitrate (GTN), arahan yang perlu diberi
kepada pesakit ialah
A. Telan ubat bersama segelas air suam
B. Menguyah ubat tersebut sebelum menelan
C. Biarkan ubat itu larut perlahan-lahan di bawah lidah
D. Ambil makanan selepas setengah jam menelan ubat
15.Tujuan utama pemerhatian rapi terhadap pesakit acute myocardial
infarction yang sedang menerima infuse streptokinase adalah untuk
mengesan
A. Takipnea
B. Pendarahan
C. Fluid overload
D. Extension of infarction
16.Komplikasi utama yang menyebabkan kadar mortality yang tinggi myocardial
infarction adalah disebabkan oleh
A. Dysrhytmias
B. Pulmonary edema
C. Left ventricle failure
D. Aortic aneurysm
17.Edema yang dialami oleh pesakit congestive cardiac failureadalah disebabkan
oleh
A Peningkatan tekanan vena
B Kekurangan serum albumin
C Ketelapan membarn sel-sel
D Kehilangan fungsi injap vena
18.Tindakan perawatan kepada pesakit kegagalan jantung yang mengalami
edema di kaki
A. Pakaikan supportive stockinet di kaki
B. Beri sokongan pada kaki semasa mengubah baringan
C. Galakkan pesakit melakukan senaman isometric kaki
D. Tinggikan kaki pesakit ke paras jantung semasa rehat
19.Tujuan utama merawat pesakit yang mengalami congestive cardiac failure
dalam posisi Fowler’s ialah
A. Meningkatkan preload
B. Meningkatkan tensi oksigen
C. Mengurangkan retensi cecair
D. Mengurangkan venous return
20.Pesakit kegagalan jantung kongestif mengeluarkan kahak yang berbuih dan
berdarah akibat
A. Infeksi di paru-paru
B. Kongesi kapilari pulmonary
C. Kesempitan salur pernafasan
D. Penyekatan salur pernafasan
21.Kegagalan penutupan foramen ovale selepas bayi dilahirkan akan
mengakibatkan
A. Tetralogy of Fallot
B. Atrial septal defect
C. Pulmonary stenosis
D. Patent ductus arteriosus
22.Tanda yang ditunjukkan oleh pesakit yang mengalami gangguan vascular
arteri periferi ialah
A. Edema di kedua-dua belah kaki
B. Kadar nadi menurun di ekstremiti
C. Kongesi vena di bahagian distal kaki
D. Sakit kejang pada otot-otot betis dan kaki
23.May adalah seorang penghidap infektif endokarditis. Beliau berisiko mengalami
cerebrovascular accident akibat
A. Iskemia
B. Aneurysm
C. Embolism
D. Pendarahan
24.Tab. Captopril yang dipreskripsikan kepada pesakit hypertensi boleh
menurunkan tekanan darah dengan
A. Mengurangkan isipadu intravascular
B. Menyekat beta-receptor di salur darah periferi
C. Mengakibatkan relaksasi otot-otot licin vascular
D. Mengurangkan pertukaran angiotensin I ke angiotensin II
25.Tujuan suntikan morphine diberi kepada pesakit myocardial infarction ialah
A. Mengelakan disritma jantung
B. Menguatkan kotraktiliti jantung
C. Mengurangkan kesakitan dan kegelisahan
D. Meningkatkan pengaliran darah ke myocardium
26.Pesakit yang mengalami ‘pernicious anemia’ selepas pembedahan gastrectomy
adalah disebabkan kekurangan
A. Zat besi
B. Asid folik
C. Vitamin B12
D. Asid hidroklorik
27.Angina pectoris berlaku disebabkan oleh
A. Nekrosis tisu miokardium
B. Kekurangan oksigen miokardium
C. Kekurangan keluaran kardiak
D. Kelemahan kotraktiliti kardiak
28.Disritmia bermaksud keabnormalan
A. Struktur jantung
B. Perfusi jantung
C. Kontraktiliti jantung
D. Fungsi injap jantung
29.Demam rheumatic akut boleh mengakibatkan kerosakan pada
A. Septum jantung
B. Miokardium
C. Endokardium
D. Injap jantung
30.Tujuan intervensi bagi deep vein thrombosis ialah untuk
A. Mencegah pendarahan
B. Mengelak kesekatan vena
C. Mengurangkan kejadian iskemia
D. Mengelak embolisme pulmonari
31.Tujuan pemberian streptokinase pada pesakit yang mengalami infeksi
miokardium
A. Mencegah pembekuan darah
B. Menambah kotraktiliti jantung
C. Menghancur bekuan darah
D. Mengawal arrhythmia
32.Selepas serangan infarksi miokardium, suhu badan pesakit meningkat dalam
masa 24-48 jam kerana
A. Reaksi terhadap tisu nekrosis
B. Sakit dada yang teruk
C. Mendapa endokarditis bacteria
D. Mendapat infeksi nosokomial
33.Sebelum pemberian digoxin kepada pesakit, jururawat perlu menilai
A. Denyutan apeks
B. Kadar nadi
C. Tekanan darah
D. Kadar pernafasan
34.Pemberian cecair berlebihan melalui infuse intravena bagi pesakit yang
mengalami kegagalan jantung kongestif boleh mengakibatkan
A. Ascitis
B. Hiperkalemia
C. Hipernatremia
D. Edema pulmonari
35.Keadaan ini sering dikaitkan dengan masalah arteri koronari
A. Polisitemia
B. Arteriosklerosis
C. Embolisme
D. Anemia
36.Pesakit yang mengalami kegagalan jantung kongestif mengeluarkan kahak
yang bercampur darah kerana
A. Jangkitan paru-paru
B. Pendarahan gusi
C. Kecederaan kapilari
D. Hemoptesis
37.Pesakit yang mengalami endokarditis boleh mendapatkan strok kerana
A. Pendarahan
B. Embolisme
C. Iskemia
D. Aneurisme
38.Berikut adalah nasihat yang anda beri kepada seorang pesakit yang mengalami
hipertensi
A. Makan ubat mengikut masa dan kurangkan dos bila tekanan darah menurun
B. Makan makanan seimbang mengikut kegemaran sendiri
C. Mengenalpasti dan elak punca stress
D. Lakukan aktiviti harian tanpa had
39.Pesakit yang menghidapi jaundis obstruktif mudah berdarah kerana
kekurangan bahan ini
A. Bilirubin
B. Vitamin B12
C. Vitamin K
D. Globulin
40.Posisi yang sesuai bagi pesakit yang mengalami CVA ( cerebral vascular
accident) dengan tekanan intrakranium yang tinggi ialah
A. Rekumben
B. Pron
C. Tinggikan bahagian kaki
D. Tinggikan bahagian kepala
41.Tindakan perawatan serta merta terhadap pesakit yang mengalami tekanan
darah tinggi 170/110 mmHg semasa kemasukan wad
A. Beri ubat hipertensi
B. Beri sokongan emosi
C. Rehatkan pesakit
D. Laporkan kepada doctor
42.Manifestasi klinikal spesifik infektif endokarditis
A. Eritma
B. Petekia
C. Perdarahan gusi
D. Murmur
43.Bekuan darah yang menyekat pengaliran vena dikenali sebagai
A. Varicose vein
B. Thrombosis vena
C. Thrombophlebitis
D. Embolism
44.Kanak-kanak yang menghidapi ‘Tetralogy of Falot’ akan menunjukkan satu ciri
klinikal berikut
A. Batuk
B. Edema
C. Sinosis
D. Demam
45.Tujuan rehat penting bagi pesakit yang mengalami kegagalan jantung
kongestif untuk
A. Mengelakan edema
B. Menggalakan selera makan
C. Mengurangkan bebanan ke atas jantung
D. Menggalakan kontraktiliti jantung
46.Bunyi pertama ‘lubb’ yang didengar semasa auscultation dengan stethoscope
dihasilkan oleh penutupan
A. Injap mitral
B. Injap aorta
C. Injap-injap semilunar
D. Injap-injap atrioventrikular
47.Antara berikut yang manakah dua (2) vena utama yang bercantum membentuk
vena portal hepatik?
A. Vena mesentrik superior dan vena splenik
B. Vena splenik dan vena gastroepiploic kanan
C. Vena mesentrik superior dan mesentrik inferior
D. Vena mesentrik inferior dan vena gastroepiploic kanan
48.Duktus limfatik kanan dan duktus torasik mengalir secara lansung ke
A. Cistern chili
B. Subclavian veins
C. Superior vena cava
D. Axillary lymph nodes
49.Semasa infeksi di system saraf, bilangan sel yang akan meningkat ialah
A. Astrosit
B. Microglia
C. Ependima
D. Oligodendrosit
50.Tindakan ubat Captopril (capoten) yang dipreskripsikan kepada pesakit
hipertensi ialah untuk
A. Menghalang rembesan aldosteron
B. Merelakskan otot licin salur darah
C. Menyekat penyerapan semula sodium
D. Mengurangkan spasma ke otot licin salur darah
51.Anemia boleh berlaku selepas gastrektomi separa kerana
A. Kekurangan factor intrinsik
B. Saiz gaster sudah menjadi kecil
C. Penghasilan jus gaster berkurangan
D. Kelambatan pencernaan zat makanan
52.Ubat untuk rawatan angina
A. Morphine
B. Betaloc
C. Streptokinase
D. Glycerin trinitrate
53.Berikut adalah komplikasi infarksi miokardium KECUALI
A. Disritmia
B. Edema pulmonari
C. Kegagalan kardiak
D. Peningkatan keluaran kardiak
54.Kesan hipertensi kronik ke atas jantung adalah
A. Stenosis injap mitral
B. Hipertrofi myocardium
C. Penurunan keluaran kahak
D. Peningkatan keluaran kahak
55.Gangguan dalam konduksi jantung akan mengakibatkan
A. Aritmia
B. Hipotensi
C. Ateriosklerosis
D. Edema pulmonari
56.Denyutan dalam sistem konduksi jantung akan mengakibatkan
A. Fluter atrium
B. Ritma sinus
C. Aritmia sinus
D. Fibrilasi atrium
57.Kesan kegagalan jantung kanan ialah
A. Angina pectoris
B. Edema sistemik
C. Edema pulmonari
D. Infarksi miokardium
58.Langkah pertama dalam resusitasi kardiopulmonari ialah
A. Memeriksa nadi carotid
B. Membuka saluran
C. Memulih denyutan jantung
D. Memulakan kompresi jantung
59.Kompresi jantung hanya dilakukan keatas jantung mangsa selepas menentukan
ketiadaan nadi
A. Radial
B. Brakial
C. Carotid
D. Temporal
60.Tujuan pemberian makanan lembut kepada pesakit infarksi miokardium akut
adalah untuk
A. Mengelakkan muntah
B. Merehatkan sistem pencernaan
C. Mengurangkan beban jantung
D. Mengelakkan sembelit
Part 3
1. The nurse is preparing a teaching plan for a 45-year-old client recently diagnosed
with type 2 diabetes mellitus. What is the first step in this process?
a. Establish goals.
b. Choose video materials and brochures.
c. Assess the client’s learning needs.
d. Set priorities of learning needs.
2. A loading dose of digoxin (Lanoxin) is given to a client newly diagnosed with atrial
fibrillation. The nurse begins instructing the client about the medication and the importance
of monitoring the heart rate. An expected outcome of the education program will be:
a. a return demonstration of palpating the radial pulse.
b. a return demonstration of how to take the medication.
c. verbalization of why the client has atrial fibrillation.
d. verbalization of the need for the medication.
3. A multigravida client is scheduled for a percutaneous umbilical blood sampling (PUBS)
procedure. The nurse instructs the client that this procedure is useful for diagnosing which of
the following?
a. Twin pregnancies.
b. Fetal lung maturation.
c. Rh disease.
d. Alpha-Fetoprotein level.
4. Which of the following is a side effect of vancomycin (Vancocin) and needs to be reported
promptly?
a. Vertigo.
b. Tinnitus.
c. Muscle stiffness.
d. Ataxia.
5. Which of the following statements indicates that the client with a peptic ulcer
understands the dietary modifications he will need to follow at home?
a. “I should eat a bland, soft diet.”
b. “It is important to eat six small meals a day.”
c. “I should drink several glasses of milk a day.”
d. “I should avoid alcohol and caffeine.”
6. The client with a nasogastric tube begins to complain of abdominal distention. Which of
the following measures should the nurse implement first?
a. Call the physician.
b. Irrigate the nasogastric tube.
c. Check the function of the suction equipment.
d. Reposition the nasogastric tube.
7. A male client has been diagnosed as having a low sperm count during infertility studies.
After instructions by the nurse about some causes of low sperm counts, the nurse
determines that the client needs further instructionswhen he says low sperm counts may be
caused by which of the following?
a. Varicocele.
b. Frequent use of saunas.
c. Endocrine imbalances.
d. Decreased body temperature.
8. The nurse assesses a client and notes puffy eyelids, swollen ankles, and crackles at both
lung bases. The nurse understands that these clinical findings are most specifically
associated with fluid excess in which of the following compartments?
a. Interstitial compartment.
b. Intravascular compartment.
c. Extracellular compartment.
d. Intracellular compartment.
9. An expected physiologic response to a low potassium level is
a. cardiac dysrhythmias.
b. hyperglycemia.
c. hypertension.
d. increased energy.
10. When teaching unlicensed assistive personnel (UAP) about the importance of
handwashing in preventing disease, the nurse makes which of the following statements?
a. “It is not necessary to wash your hands as long as you use gloves.”
b. “Handwashing is the best method for preventing cross-contamination.”
c. “Waterless commercial products are not effective for killing organisms.”
d. “The hands do not serve as a source of infection.”
11. The nurse is performing Leopold maneuvers on a woman who is in her eighth month of
pregnancy. The nurse is palpating the uterus at the symphysis pubis area. Which of the
following maneuvers is the nurse performing?
a. First maneuver.
b. Second maneuver.
c. Third maneuver.
d. Fourth maneuver.
12. A client in a cardiac rehabilitation program states that he would like to make sure he is
eating the right foods to ensure adequate endurance on the treadmill. Which of the
following nutrients is most helpful for promoting endurance during sustained activity?
a. Protein.
b. Carbohydrate.
c. Fat.
d. Water
13. A client’s chest tube is connected to a chest tube drainage system with a water seal. The
nurse notes that the fluid in the water-seal column is fluctuating with each breath that the
client takes. The fluctuation means that
a. there is an obstruction in the chest tube.
b. the client is developing subcutaneous emphysema.
c. the chest tube system is functioning properly.
d. there is a leak in the chest tube system.
14. A client with diabetes is explaining to the nurse how she will care for her feet at home.
Which statement indicates that the client understands proper foot care?
a. “When I injure my toe, I will plan to put iodine on it.”
b. “I should inspect my feet at least once a week.”
c. “I do not plan to wear shoes while I am in the house.”
d. “It is important to dry my feet carefully after my bath.”
15. The nurse assesses a client with diverticulitis and suspects peritonitis when which of the
following symptoms is noted?
a. Hyperactive bowel sounds.
b. Rigid abdominal wall.
c. Explosive diarrhea.
d. Excessive flatulence.
16. When assessing a client, which risk factors would lead the nurse to suspect that the
client has pancreatitis? Select all that apply.
a. Excessive alcohol use.
b. Gallstones.
c. Abdominal trauma.
d. Hyperlipidemia with excessive triglycerides.
17. When performing chest percussion on a child, which of the following techniques would
the nurse use?
a. Firmly but gently striking the chest wall to make a popping sound.
b. Gently striking the chest wall to make a slapping sound
c. Percussing over an area from the umbilicus to the clavicle.
d. Placing a blanket between the nurse’s hand and the child’s chest.
18. The nurse walks into the room of a client who has a “Do Not Resuscitate” order and finds
the client without a pulse, respirations, or blood pressure. What is the most appropriate
action?
a. Stay in the room and notify the nursing team for assistance.
b. Push the emergency alarm to call a code.
c. Dial the hospital phone number for a code.
d. Pull the curtain and leave the room.
19. A client is trying to lose weight at a moderate pace. If the client eliminates 1000 calories
per day from his normal intake, how many pounds would he lose in 1 week?
a. 1 pound.
b. 2 pounds.
c. 3 pounds.
d. 4 pounds.
20. A nulliparous client calls the clinic and tells the nurse that she forgot to take her oral
contraceptive this morning. Which of the following would the nurse instruct the client to do?
a. Take the medication immediately.
b. Restart the medication in the morning.
c. Use another form of contraception for 2 weeks.
d. Take two pills tonight before bedtime.
Part 1
1. A client with HIV and AIDS confides that he is homosexual and his employer does
not know his HIV status. The nurse’s best response to him is:
A. “Would you like me to help you tell them?”
B. “The information you confide in me is confidential.”
C. “I must share this information with your family.”
D. “I must share this information with your employer.”
2. The mother of a child with bronchial asthma tells the nurse that the child wants a
pet. Which of the following pets should the nurse tell the mother is most
appropriate?
A. Cat.
B. Fish.
C. Gerbil.
D. Canary.
3. An elderly client is being admitted to same-day surgery for cataract extraction.
The client has several diamond rings. The nurse should explain to the client
that
A. her rings will be taped before the surgery.
B. she will sign a valuables envelope that will be placed in a safe.
C. the rings will be locked in the narcotics box.
D. the nursing supervisor will hold onto the rings during the surgery.
4. When an infant resumes taking oral feedings after surgery to correct
intussusception, the parents comment that the child seems to suck on the
pacifier more since the surgery. The nurse explains that sucking on a
pacifier
A. provides an outlet for emotional tension.
B. indicates readiness to take solid foods.
C. indicates intestinal motility.
D. is an attempt to get attention from the parents.
5. A 22-year-old client is brought to the emergency department with his fiancée
after being involved in a serious motor vehicle accident. His GlasgowComa
Scale score is 7 and he demonstrates evidence of decorticate posturing.
Which of the following would be appropriate for obtaining permission to
place a catheter for intracranial pressure (ICP) monitoring?
A. The nurse will obtain a signed consent from the client’s fiancée because he is of legal
age and they are engaged to be married.
B. The physician will get a consultation from one other physician and proceed
with placement of the ICP catheter until the family arrives to sign the consent.
C. Two nurses will receive a verbal consent by telephone from the client’s next of
kin before inserting the catheter.
D. The physician will document the emergency nature of the client’s condition and
that an ICP catheter for monitoring was placed without a consent.
6. A 68-year-old client’s daughter is asking about the follow-up evaluation for her
father after his pneumonectomy for primary lung cancer. The nurse’s best
response is which of the following?
A. “The usual follow-up is chest x-ray and liver function tests every 3 months.”
B. “The follow-up for your father will be a chest x-ray and a computed
tomography (CT) scan of the abdomen every year.”
C. “No follow-up is needed at this time.”
D. “The follow-up for your father will be a chest x-ray every 6 months.”
7. The nurse is preparing to administer blood to an otherwise healthy client who
requires postoperative blood replacement. The nurse is aware that the
blood administration set must include
A. a micron mesh filter.
B. a nonfiltered administration blood set.
C. a special leukocyte-poor filter.
D. a microdrip administration set.
8. Under which circumstance may a nurse communicate medical information
without the client’s consent?
A. When certifying the client’s absence from work.
B. When requested by the client’s family.
C. When treating clients who have a sexually transmitted disease (STD).
D. When ordered by another physician.
9. During the health history interview, which of the following strategies is the most
effective for the nurse to use to help clients feel that they have an active role
in their health care?
A. Ask clients to complete a questionnaire.
B. Provide clients with written instructions.
C. Ask clients for their description of events and for their views concerning past
medical care.
D. Ask clients if they have any questions.
10. A client with severe major depression states, “My heart has stopped and my
blood is black ash.” The nurse interprets this statement to be evidence of
which of the following?
A. Hallucination.
B. Illusion.
C. Delusion.
D. Paranoia.
11. When a client wants to read his chart, the nurse should
A. call the doctor to obtain permission.
B. give the client the chart and answer questions for him.
C. tell the client that he can read the chart when the doctor makes rounds.
D. ask the client what he wants to know and answer those questions without
giving him the chart.
12. A client who has a fractured leg has been instructed to ambulate without weight
bearing on the affected leg. The nurse evaluates that the client is
ambulating correctly if she uses which of the following crutch-walking
gaits?
A. Two-point gait.
B. Four-point gait.
C. Three-point gait.
D. Swing-to gait.
13. A client with major depression states, “Life isn’t worth living anymore. Nothing
matters.” Which of the following responses by the nurse would be best?
A. “Are you thinking about killing yourself?”
B. “Things will get better, you know.”
C. “Why do you think that way?”
D. “You shouldn’t feel that way.”
14. A client with bipolar 1 disorder has been prescribed olanzapine (Zyprexa) 5 mg
two times a day and lamotrigine (Lamictal) 25 mg two times a day. Which
of the following adverse effects would the nurse report to the physician
immediately? Select all that apply.
A. Rash.
B. Nausea.
C. Hyperthemia.
D. Muscle rigidity.
15. A client is prescribed atropine, 0.4 mg IM. The atropine vial is labeled 0.5
mg/mL. How many milliliters should the nurse plan to administer?
A. 0.8.
B. 0.4
C. 8
D. 0.5
16. A multiparous client tells the nurse that she is using medroxyprogesterone
(Depo-Provera) for contraception. The nurse instructs the client to increase
her intake of which of the following?
A. Folic acid.
B. Vitamin C.
C. Magnesium
D. Calcium.
17. Which of the following statements made by a woman in the first trimester are
consistent with this stage of pregnancy? Select all that apply.
A. “My husband told his friends we will have to give up the mustang for a minivan.”
B. “Oh my, how did this happen? I don’t need this now.”
C. “I can’t wait to see my baby. Do you think it will have my blond hair and blue
eyes?”
D. “I wonder how it will feel to buy maternity clothes and be fat.”
18. The nurse is teaching a client about topical gentamicin sulfate (Garamycin).
Which of the following comments by the client indicates the need for
additional teaching?
A. “I will avoid being out in the sun for long periods.”
B. “I should stop applying it once the infected area heals.”
C. “I’ll call the physician if the condition worsens.”
D. “I should apply it to large open areas.”
19. A client has been taking imipramine (Tofranil) for his depression for 2 days. His
sister asks the nurse, “Why is he still so depressed?” Which of the following
responses by the nurse would be most appropriate?
A. “Your brother is experiencing a very serious depression.”
B. “I’ll be sure to convey your concern to his physician.”
C.”It takes 2 to 4 weeks for the drug to reach its full effect.”
D. “Perhaps we’ll need to change his medication.”
20. Which interventions would the nurse use to assist the client with grandiose
delusions? Select all that apply.
A. Accepting the client while not arguing with the delusion.
B. Focusing on the feelings or meaning of the delusion.
C. Focusing on events and topics based in reality.
D. Confronting the client’s beliefs.
NCLEX Comprehensive Exam Part 1
Questions
Answer B. The nurse is responsible for maintaining confidentiality of this
disclosure by the client.
Answer B. Pets are discouraged when parents are trying to allergy-proof a
home for a child with bronchial asthma, unless the pets are kept outside. Pets
with hair or feathers are especially likely to trigger asthma attacks. A fish
would be a satisfactory pet for this child, but the parents should be taught to
keep the fish tank clean to prevent it from harboring mold
Answer B. Under the policy for valuables, the nurse documents the description
on an envelope with the client, the client and nurse sign the envelope, and the
valuables envelope is locked in the safe. The other options increase the risk of
loss or damage to the client’s valuables.
Answer A. Sucking provides the infant with a sense of security and comfort. It
also is an outlet for releasing tension. The infant should not be discouraged
from sucking on the pacifier. Fussiness and irritability after feeding may
indicate that the infant’s appetite is not satisfied. Sucking is not manipulative
in the sense of seeking parental attention.
Answer D. In a life-threatening emergency where time is of the essence in
saving life or limb, a consent is not required. This client has a Glasgow Coma
Scale score of 7, which means he is comatose. The client has deteriorated to a
level where he cannot be aroused; withdraws in a purposeless manner from
painful stimuli; exhibits decorticate posturing; and may or may not have brain
stem reflexes intact. The placement of the ICP monitor is crucial to determine
cerebral blood flow and prevent herniation. The client’s fiancée can not sign
his consent because, until she is his wife or has designated power of attorney,
she is not considered his next of kin. The physician should insert the catheter
in this emergency. He does not need to get a consultation from one other
physician. When a consent is needed for a situation that is not a true
emergency, two nurses can receive a verbal consent by telephone from the
client’s next of kin.
Answer D. Follow-up generally involves semiannual chest radiographs.
Recurrence usually occurs locally in the lungs and may be identified on chest
radiographs. Follow-up after cancer treatment is an important component of
the treatment plan. Serum markers (eg, liver function tests) have not been
shown to detect recurrence of lung cancer. There are no data to support the
need for an abdominal CT scan.
Answer A. All blood products should be administered through a micron mesh
filter. Blood is never administered without a filter. Leukocytes can be removed
by using leukocyte-poor filters, and this is recommended to decrease reactions
in clients such as hemophiliacs who require frequent transfusions. Blood is too
concentrated to administer through a microdrip set.
Answer C. STDs are communicable diseases that must be reported. The nurse is
responsible for reporting these diseases to the appropriate public health
agency, and to otherwise maintain the client’s confidentiality. The client’s
family cannot request release of medical information without the client’s
consent. A physician’s order is not a substitute for a client’s consent to release
medical information in the absence of a communicable disease.
Answer C. One of the best strategies to help clients feel in control is to ask
them their view of situations, and to respond to what they say. This technique
acknowledges that clients’ opinions have value and relevance to the interview.
It also promotes an active role for clients in the process. Use of a questionnaire
or written instructions is a means of obtaining information but promotes
a passiveclient role. Asking whether the client has questions encourages client
participation, but alone it does not acknowledge the client’s views.
Answer C. A client with severe depression may experience symptoms of
psychosis such as hallucinations and delusions that are typically mood
congruent. The statement, “My heart has stopped and my blood is black ash,” is
a mood-congruent somatic delusion. A delusion is a firm, false, fixed belief that
is resistant to reason or fact. A hallucination is a false sensory perception
unrelated to external stimuli. An illusion is a misinterpretation of a real
sensory stimulus. Paranoia refers to suspiciousness of others and their actions.
Answer B. The client should be allowed to see his chart. As a client advocate,
the nurse should answer questions for the client. The nurse helps the client
understand that he is a primary partner in the health team. The Bill of Rights
for Patients has existed since the 1960s, and every client should be aware of
this document. The doctor should not need to give permission for the client to
see his chart. As a client advocate, the nurse should not make excuses to put
the client off in regard to seeing his chart.
Answer C. The three-point gait, in which the client advances the crutches and
the affected leg at the same time while weight is supported on the unaffected
extremity, is the appropriate gait of choice. This allows for non–weight
bearing on the affected extremity. The two-point, four-point, and swing-to
gaits require some weight bearing on both legs, which is contraindicated for
this client.
Answer A. When the client verbalizes that life isn’t worth living anymore, the
nurse needs to ask the client directly about suicide by saying, “Are you
thinking about killing yourself?” Asking directly does not provoke suicide but
conveys concern, understanding, and the worth of the client. Often, the client
experiences a sense of relief that someone finally hears him. It also helps the
nurse plan responsible care by identifying the client who is at risk for suicide.
The nurse would then evaluate the seriousness of the suicidal ideation by
inquiring about the intent and plan. Stating, “Things will get better,” offers
hope too soon without first evaluating the intent of the suicidal ideation.
Asking, “Why do you think that way,” implies a lack of understanding and
knowledge on the part of the nurse. Major depression usually is endogenous
and biochemically based. Therefore, the client may not know why he doesn’t
want to live. Saying, “You shouldn’t feel that way,” admonishes the client,
decreases self-worth, and conveys a lack of understanding.
Answer A, C and D. Lamotrigine (Lamictal), an antiepileptic, is used as a mood
stabilizer for clients with bipolar disorder and has been found to be effective
for the depressive phase of bipolar disorder. Common adverse effects are
dizziness, headache, sedation, tremor, nausea, vomiting, and ataxia. The
development of a rash needs to be reported and evaluated by the physician
because it could indicate the start of a severe systemic rash known as Stevens-
Johnson syndrome, a toxic epidermal necrolysis, which would necessitate the
discontinuance of lamotrigine. Hyperthermia in conjunction with muscle
rigidity suggests the development of the neuroleptic malignant syndrome, a
life-threatening complication associated with olanzapine.
Answer A. 0.4 mg/x = 0.5 mg/1 mL 0.4 = 0.5x 0.4/0.5 = x 0.8 ml = x.
Answer D. The nurse should instruct the client to increase her intake of
calcium because there is a slight increase in the risk of osteoporosis with this
medication. Weight-bearing exercises are also advised. The drug may also
impair glucose tolerance in women who are at risk for diabetes.
Answer A, B and D. The first trimester is when the couple works through the
psychological task of accepting the pregnancy. These statements describe the
client and her partner coping with the pregnancy, how it feels, and how it will
impact their lives. The feelings include pleasure, excitement and ambivalence.
Wondering what the baby will look like and planning for the baby’s room
occur later in the pregnancy.
Answer D. The aminoglycoside antibiotic gentamicin sulfate (Garamycin)
should not be applied to large denuded areas because toxicity and systemic
absorption are possible. The nurse should instruct the client to avoid excessive
sun exposure because gentamicin sulfate can cause photosensitivity. The client
should be instructed to apply the antibiotic cream or ointment for only the
length of time prescribed, because a superinfection can occur from overuse.
The client should contact the physician if the condition worsens after use.
Answer C. The nurse needs to inform the sister that there is a lag time of 2 to 4
weeks before a full clinical effect occurs with the drug. The nurse should let
her know that her brother will gradually get better and symptoms of
depression will improve. Telling the sister that her brother is experiencing a
very serious depression does not give the sister important information about
the medication. Additionally, this statement may cause alarm and anxiety.
Conveying the sister’s concern to the physician does not provide her with the
necessary information about the client’s medication. Telling the sister that the
client’s medication may need to be changed is inappropriate because a full
clinical effect occurs after 2 to 4 weeks.
Answer C. For the client with grandiose delusions, the nurse would accept the
client but not argue with the delusion to build trust and the client’s self-
esteem. Focusing on the underlying feeling or meaning of the delusion helps to
meet the client’s needs. Focusing on events and topics based in reality distracts
the client from the delusional thinking. Confronting the client’s delusions or
beliefs can lead to agitation in the client and the need to cling to the grandiose
delusion to preserve self-esteem. Interacting with the client only when he is
based in reality ignores the client’s needs and therapeutic nursing
intervention.
Part 2
1. A multigravida client visiting the prenatal clinic at 16 weeks’ gestation
exhibits facial swelling, a brownish vaginal discharge, and fundal height of 22
cm. The client’s blood pressure is 160/90 mm Hg and her pulse is 80 bpm. The
nurse interprets these findings as suggestive of which of the following?
a. Placenta previa.
b. Fetal anemia.
c. Multifetal pregnancy.
d. Gestational trophoblastic disease.
2. Which of the following responses would be most helpful for a client who is
euphoric, intrusive, and interrupts other clients engaged in conversations to
the point where they get up and leave or walk away?
a. “When you interrupt others, they leave the area.”
b. You are being rude and uncaring.”
c. “You should remember to use your manners.”
d. “You know better than to interrupt someone.”
3. The nurse coordinates with the laboratory staff to have the gentamicin
trough serum level drawn. At what time should the blood be drawn in relation
to the administration of the intravenous dose of gentamicin sulfate
(Garamycin)?
a. 2 hours before the administration of the next intravenous dose.
b. 3 hours before the administration of the next intravenous dose.
c. 4 hours before the administration of the next intravenous dose.
d. Just before the administration of the next intravenous dose.
4. Older adults with known cardiovascular disease must balance which of the
following measures for optimum health?
a. Diet, exercise, and medication.
b. Stress, hypertension, and pain.
c. Mental health, diet, and stress.
d. Social events, diet, and smoking.
5. A 4-year-old is brought to the emergency department with sudden onset of a
temperature of 103°F (39.5°C), sore throat, and refusal to drink. The child will
not lie down and prefers to lean forward while sitting up. Which of the
following would the nurse do next?
a. Give 600 mg acetaminophen (Tylenol) per rectum as ordered.
b. Inspect the child’s throat for redness and swelling.
c. Have an appropriate-sized tracheostomy tube readily available.
d. Obtain a specimen for a throat culture.
6. Assessment of a client taking lithium reveals dry mouth, nausea, thirst, and
mild hand tremor. Based on an analysis of these findings, which of the
following would the nurse do next?
a. Hold the lithium and obtain a stat lithium level to determine therapeutic
effectiveness.
b. Continue the lithium and immediately notify the physician about the
assessment findings.
c. Continue the lithium and reassure the client that these temporary side
effects will subside.
d. Hold the lithium and monitor the client for signs and symptoms of
increasing toxicity.
7. A client asks the nurse how long she will have to take her medicine for
hypothyroidism. The nurse’s response is based on the knowledge that
a. lifelong daily medicine is necessary.
b. the medication is expensive, and the dose can be reduced in a few months.
c. the medication can be gradually withdrawn in 1 to 2 years.
d. the medication can be discontinued after the client’s thyroid-stimulating
hormone (TSH) level is normal.
8. Assessment of which of the following clients would lead the nurse to expect
the physician to order an adjustment in lithium dosage?
a. A client who continues work as a computer programmer.
b. A client who attends college classes.
c. A client who is now able to care for his or her children.
d. A client who is beginning training for a tennis team.
9. A client admitted with a gastric ulcer has been vomiting bright red blood.
His hemoglobin is 5.11 g/dL, and his blood pressure is 100/50 mm Hg. The
client and the family state that their religious beliefs do not support the use of
blood products and refuse blood transfusions as a treatment for the bleeding.
The nurse would expect that the next step in the treatment plan would be to
a. discontinue all measures.
b. notify the hospital attorney.
c. attempt to stabilize the client through the use of fluid replacement.
d. give enough blood to keep the client from dying.
10. The parents of a child with cystic fibrosis express concern about how the
disease was transmitted to their child. The nurse would explain that
a. a disease carrier also has the disease.
b. two parents who are carriers may produce a child who has the disease.
c. a disease carrier and an affected person will never have children with the
disease.
d. a disease carrier and an affected person will have a child with the disease.
11. A client with angina shows the nurse her nitroglycerin (Nitrostat) that she
is carrying in a plastic bag in her pocket. The nurse instructs the client that
nitroglycerin should be kept
a. in the refrigerator.
b. in a cool, moist place.
c. in a dark container to shield from light.
d. in a plastic bag where it is readily available
12. The nurse caring for client on the telemetry unit is able to determine that
the client is in sinus bradycardia by recognizing which characteristics? Select
all that apply.
a. P wave present.
b. Ventricular rate of 50 beats per minute (bpm).
c. Atrial rate of 120 bpm.
d. PR interval ranging from 0.12 to 0.20.
13. When teaching a client with bipolar disorder, mania, who has started to
take valproic acid (Depakene) about possible side effects of this medication,
the nurse would include which of the following in the teaching plan?
a. Increased urination.
b. Slowed thinking.
c. Sedation.
d. Weight loss.
14. An infant is born with facial abnormalities, growth retardation, mental
retardation, and vision abnormalities. These abnormalities are probably
caused by maternal
a. alcohol consumption
b. vitamin B6 deficiency.
c. vitamin A deficiency.
d. folic acid deficiency.
15. Nonsteroidal anti-inflammatory drugs (NSAIDs) are frequently used in the
treatment of musculoskeletal conditions. It is important for the nurse to
remind the client to
a. take NSAIDs at least three times per day.
b. exercise the joints at least 1 hour after taking the medication.
c. take antacids 1 hour after taking NSAIDs.
d. take NSAIDs with food.
16. The nurse would suspect that the client taking disulfiram (Antabuse)
therapy has ingested alcohol when the client exhibits which of the following
symptoms?
a. Sore throat and muscle aches.
b. Nausea and flushing of the face and neck.
c. Fever and muscle soreness.
d. Bradycardia and vertigo.
17. The nurse holds the gauze pledget against an intramuscular injection site
while removing the needle from the muscle. This technique helps to
a. seal off the track left by the needle in the tissue.
b. speed the spread of the medication in the tissue.
c. avoid the discomfort of the needle pulling on the skin.
d. prevent organisms from entering the body through the skin puncture.
18. A client whose condition remains stable after a myocardial infarction
gradually increases his activity. Which the following conditions should the
nurse assess to determine whether the activity is appropriate for the client?
a. Edema.
b. Cyanosis.
c. Dyspnea.
d. Weight loss
19. When a client with alcohol dependency begins to talk about not having a
problem with alcohol, the nurse would use which of the following approaches?
a. Questioning the client about how much alcohol she drinks.
b. Confronting the client with the fact that she was intoxicated 2 days ago
c. Pointing out how alcohol has gotten her into trouble.
d. Listening to what the client states and then asking her how she plans to stay
sober.
20. Which of the following correctly describes Medicaid?
a. A program designed to assist ill, low-income older adults.
b. A federal insurance program for pregnant women.
c. A joint federal–state program for low-income persons.
d. A program administered by health maintenance organizations.
1. Which individual is at greatest risk for developing hypertension?
A) 45 year-old African American attorney
B) 60 year-old Asian American shop owner
C) 40 year-old Caucasian nurse
D) 55 year-old Hispanic teacher
2. A child who ingested 15 maximum strength acetaminophen tablets 45
minutes ago is seen in the emergency department. Which of these orders
should the nurse do first?
A) Gastric lavage PRN
B) Acetylcysteine (mucomyst) for age per pharmacy
C) Start an IV Dextrose 5% with 0.33% normal saline to keep vein open
D) Activated charcoal per pharmacy
3. Which complication of cardiac catheterization should the nurse monitor for
in the initial 24 hours after the procedure?
A) angina at rest
B) thrombus formation
C) dizziness
D) falling blood pressure
4. A client is admitted to the emergency room with renal calculi and is
complaining of moderate to severe flank pain and nausea. The client’s
temperature is 100.8 degrees Fahrenheit. The priority nursing goal for this
client is
A) Maintain fluid and electrolyte balance
B) Control nausea
C) Manage pain
D) Prevent urinary tract infection
5. What would the nurse expect to see while assessing the growth of children
during their school age years?
A) Decreasing amounts of body fat and muscle mass
B) Little change in body appearance from year to year
C) Progressive height increase of 4 inches each year
D) Yearly weight gain of about 5.5 pounds per year
6. At a community health fair the blood pressure of a 62 year-old client is
160/96. The client states “My blood pressure is usually much lower.” The nurse
should tell the client to
A) go get a blood pressure check within the next 48 to 72 hours
B) check blood pressure again in 2 months
C) see the health care provider immediately
D) visit the health care provider within 1 week for a BP check
7. The hospital has sounded the call for a disaster drill on the evening shift.
Which of these clients would the nurse put first on the list to be discharged in
order to make a room available for a new admission?
A) A middle aged client with a history of being ventilator dependent for over
7 years and admitted with bacterial pneumonia five days ago
B) A young adult with diabetes mellitus Type 2 for over 10 years and
admitted with antibiotic induced diarrhea 24 hours ago
C) An elderly client with a history of hypertension, hypercholesterolemia
and lupus, and was admitted with Stevens-Johnson syndrome that morning
D) An adolescent with a positive HIV test and admitted for acute cellulitus
of the lower leg 48 hours ago
8. A client has been newly diagnosed with hypothyroidism and will take
levothyroxine (Synthroid) 50 mcg/day by mouth. As part of the teaching plan,
the nurse emphasizes that this medication:
A) Should be taken in the morning
B) May decrease the client’s energy level
C) Must be stored in a dark container
D) Will decrease the client’s heart rate
9. A 3 year-old child comes to the pediatric clinic after the sudden onset of
findings that include irritability, thick muffled voice, croaking on inspiration,
hot to touch, sit leaning forward, tongue protruding, drooling and
suprasternal retractions. What should the nurse do first?
A) Prepare the child for x-ray of upper airways
B) Examine the child’s throat
C) Collect a sputum specimen
D) Notify the healthcare provider of the child’s status
10. In children suspected to have a diagnosis of diabetes, which one of the
following complaints would be most likely to prompt parents to take their
school age child for evaluation?
A) Polyphagia
B) Dehydration
C) Bed wetting
D) Weight loss
11. A client comes to the clinic for treatment of recurrent pelvic inflammatory
disease. The nurse recognizes that this condition most frequently follows
which type of infection?
A) Trichomoniasis
B) Chlamydia
C) Staphylococcus
D) Streptococcus
12. An RN who usually works in a spinal rehabilitation unit is floated to the
emergency department. Which of these clients should the charge nurse assign
to this RN?
A) A middle-aged client who says "I took too many diet pills" and "my heart
feels like it is racing out of my chest."
B) A young adult who says "I hear songs from heaven. I need money for beer.
I quit drinking 2 days ago for my family. Why are my arms and legs jerking?"
C) An adolescent who has been on pain medications terminal cancer with an
initial assessment finding pupils and a relaxed respiratory rate of 10
D) An elderly client who reports having taken a "large crack hit" 10 minutes
prior to walking into the emergency room
13. When teaching a client with coronary artery disease about nutrition, the
nurse should emphasize
A) Eating 3 balanced meals a day
B) Adding complex carbohydrates
C) Avoiding very heavy meals
D) Limiting sodium to 7 gms per day
14. Which of these findings indicate that a pump to deliver a basal rate of 10
ml per hour plus PRN for pain break through for morphine drip is not
working?
A) The client complains of discomfort at the IV insertion site
B) The client states "I just can’t get relief from my pain."
C) The level of drug is 100 ml at 8 AM and is 80 ml at noon
D) The level of the drug is 100 ml at 8 AM and is 50 ml at noon
15. The nurse is speaking at a community meeting about personal
responsibility for health promotion. A participant asks about chiropractic
treatment for illnesses. What should be the focus of the nurse’s response?
A) Electrical energy fields
B) Spinal column manipulation
C) Mind-body balance
D) Exercise of joints
16. The nurse is performing a neurological assessment on a client post right
CVA. Which finding, if observed by the nurse, would warrant immediate
attention?
A) Decrease in level of consciousness
B) Loss of bladder control
C) Altered sensation to stimuli
D) Emotional ability
17. A child who has recently been diagnosed with cystic fibrosis is in a
pediatric clinic where a nurse is performing an assessment. Which later
finding of this disease would the nurse not expect to see at this time?
A) Positive sweat test
B) Bulky greasy stools
C) Moist, productive cough
D) Meconium ileus
18. The home health nurse visits a male client to provide wound care and finds
the client lethargic and confused. His wife states he fell down the stairs 2
hours ago. The nurse should
A) Place a call to the client’s health care provider for instructions
B) Send him to the emergency room for evaluation
C) Reassure the client’s wife that the symptoms are transient
D) Instruct the client’s wife to call the doctor if his symptoms become worse
19. Which of the following should the nurse implement to prepare a client for a
KUB (Kidney, Ureter, Bladder) radiograph test?
A) Client must be NPO before the examination
B) Enema to be administered prior to the examination
C) Medicate client with Lasix 20 mg IV 30 minutes prior to the examination
D) No special orders are necessary for this examination
20. The nurse is giving discharge teaching to a client 7 days post myocardial
infarction. He asks the nurse why he must wait 6 weeks before having sexual
intercourse. What is the best response by the nurse to this question?
A) "You need to regain your strength before attempting such exertion."
B) "When you can climb 2 flights of stairs without problems, it is generally
safe.”
C) "Have a glass of wine to relax you, then you can try to have sex."
D) "If you can maintain an active walking program, you will have less risk."
21. A triage nurse has these 4 clients arrive in the emergency department
within 15 minutes. Which client should the triage nurse send back to be seen
first?
A) A 2 month old infant with a history of rolling off the bed and has bulging
fontanels with crying
B) A teenager who got a singed beard while camping
C) An elderly client with complaints of frequent liquid brown colored stools
D) A middle aged client with intermittent pain behind the right scapula
22. While planning care for a toddler, the nurse teaches the parents about the
expected developmental changes for this age. Which statement by the mother
shows that she understands the child’s developmental needs?
A) "I want to protect my child from any falls."
B) "I will set limits on exploring the house."
C) "I understand the need to use those new skills."
D) "I intend to keep control over our child."
23. The nurse is preparing to administer an enteral feeding to a client via a
nasogastric feeding tube. The most important action of the nurse is
A) Verify correct placement of the tube
B) Check that the feeding solution matches the dietary order
C) Aspirate abdominal contents to determine the amount of last feeding
remaining in stomach
D) D) Ensure that feeding solution is at room temperature
24. The nurse is caring for a client with a serum potassium level of 3.5 mEq/L.
The client is placed on a cardiac monitor and receives 40 mEq KCL in 1000 ml
of 5% dextrose in water IV. Which of the following EKG patterns indicates to
the nurse that the infusions should be discontinued?
A) Narrowed QRS complex
B) Shortened "PR" interval
C) Tall peaked T waves
D) Prominent "U" waves
25. A nurse prepares to care for a 4 year-old newly admitted for
rhabdomyosarcoma. The nurse should alert the staff to pay more attention to
the function of which area of the body?
A) All striated muscles
B) The cerebellum
C) The kidneys
D) The leg bones
26. The nurse anticipates that for a family who practices Chinese medicine the
priority goal would be to
A) Achieve harmony
B) Maintain a balance of energy
C) Respect life
D) Restore yin and yang
27. During an assessment of a client with cardiomyopathy, the nurse finds that
the systolic blood pressure has decreased from 145 to 110 mm Hg and the
heart rate has risen from 72 to 96 beats per minute and the client complains of
periodic dizzy spells. The nurse instructs the client to
A) Increase fluids that are high in protein
B) Restrict fluids
C) Force fluids and reassess blood pressure
D) D) Limit fluids to non-caffeine beverages
28. A client has a Swan-Ganz catheter in place. The nurse understands that
this is intended to measure
A) Right heart function
B) Left heart function
C) Renal tubule function
D) Carotid artery function
29. A nurse enters a client’s room to discover that the client has no pulse or
respirations. After calling for help, the first action the nurse should take is
A) Start a peripheral IV
B) Initiate closed-chest massage
C) Establish an airway
D) Obtain the crash cart
30. A client is receiving digoxin (Lanoxin) 0.25 mg. Daily. The health care
provider has written a new order to give metoprolol (Lopressor) 25 mg. B.I.D.
In assessing the client prior to administering the medications, which of the
following should the nurse report immediately to the health care provider?
A) Blood pressure 94/60
B) Heart rate 76
C) Urine output 50 ml/hour
D) Respiratory rate 16
31. While assessing a 1 month-old infant, which finding should the nurse
report immediately?
A) Abdominal respirations
B) Irregular breathing rate
C) Inspiratory grunt
D) Increased heart rate with crying
32. The nurse practicing in a maternity setting recognizes that the post
mature fetus is at risk due to
A) Excessive fetal weight
B) Low blood sugar levels
C) Depletion of subcutaneous fat
D) Progressive placental insufficiency
33. The nurse is caring for a client who had a total hip replacement 4 days
ago. Which assessment requires the nurse’s immediate attention?
A) I have bad muscle spasms in my lower leg of the affected extremity.
B) "I just can’t ‘catch my breath’ over the past few minutes and I think I am
in grave danger."
C) "I have to use the bedpan to pass my water at least every 1 to 2 hours."
D) "It seems that the pain medication is not working as well today."
34. A client has been taking furosemide (Lasix) for the past week. The nurse
recognizes which finding may indicate the client is experiencing a negative
side effect from the medication?
A) Weight gain of 5 pounds
B) Edema of the ankles
C) Gastric irritability
D) Decreased appetite
35. A client who is pregnant comes to the clinic for a first visit. The nurse
gathers data about her obstetric history, which includes 3 year-old twins at
home and a miscarriage 10 years ago at 12 weeks gestation. How would the
nurse accurately document this information?
A) Gravida 4 para 2
B) Gravida 2 para 1
C) Gravida 3 para 1
D) Gravida 3 para 2
36. The nurse is caring for a client with a venous stasis ulcer. Which nursing
intervention would be most effective in promoting healing?
A) Apply dressing using sterile technique
B) Improve the client’s nutrition status
C) Initiate limb compression therapy
D) Begin proteolytic debridement
37. A nurse is to administer meperidine hydrochloride (Demerol) 100 mg,
atropine sulfate (Atropisol) 0.4 mg, and promethizine hydrochloride
(Phenergan) 50 mg IM to a pre-operative client. Which action should the nurse
take first?
A) Raise the side rails on the bed
B) Place the call bell within reach
C) Instruct the client to remain in bed
D) D) Have the client empty bladder
38. Which of these statements best describes the characteristic of an effective
reward-feedback system?
A) Specific feedback is given as close to the event as possible
B) Staff are given feedback in equal amounts over time
C) Positive statements are to precede a negative statement
D) Performance goals should be higher than what is attainable
39. A client with multiple sclerosis plans to begin an exercise program. In
addition to discussing the benefits of regular exercise, the nurse should
caution the client to avoid activities which
A) Increase the heart rate
B) Lead to dehydration
C) Are considered aerobic
D) May be competitive
40. During the evaluation of the quality of home care for a client with
Alzheimer’s disease, the priority for the nurse is to reinforce which statement
by a family member?
A) At least 2 full meals a day is eaten.
B) We go to a group discussion every week at our community center.
C) We have safety bars installed in the bathroom and have 24 hour alarms
on the doors.
D) The medication is not a problem to have it taken 3 times a day.
Answer part 2
1. ANSWER D. Symptoms of pregnancy-induced hypertension (PIH), such as
hypertension and facial swelling, before 24 weeks’ gestation and a fundal
height larger than expected are suggestive of gestational trophoblastic disease
or hydatidiform mole. This condition occurs when the trophoblasts develop
abnormally. Ultrasound can confirm the condition. Medical management
includes evacuation of the mole and follow-up to detect any malignant
changes. Painless vaginal bleeding is suggestive of placenta previa. Fetal
anemia is often caused by Rh sensitization. Clients with multifetal pregnancies
may exhibit larger fundal heights than expected, but they usually do not have
a brownish discharge or symptoms of PIH at this stage of gestation.
2. ANSWER A. Saying, “When you interrupt others, they leave the area,” is most
helpful because it serves to increase the client’s awareness of how others view
him by giving him specific feedback about his behavior. The other statements
are punitive and authoritative, possibly threatening to the client, and likely to
increase defensiveness, decrease self-worth, and increase feelings of guilt.
3. ANSWER D. The trough serum level should be drawn just before the
administration of the next intravenous dose of gentamicin sulfate
(Garamycin).
4. ANSWER A. Health-promoting strategies for clients with a history of
cardiovascular disease require knowledge in three areas: diet, exercise, and
medication. Pain management and management of social activities are not
usually features of health promotion activities for these clients.
5. ANSWER C. The child is exhibiting signs and symptoms of possible
epiglottitis. As a result the child is at high risk for laryngospasm and airway
occlusion. Therefore, the nurse should have a tracheostomy tube and setup
readily available should the child experience an airway occlusion. Although
acetaminophen is an antipyretic, the dosage of 600 mg to be administered
rectally is too high. A typical 4-year-old weighs approximately 40 pounds. The
recommended dose is 125 mg. When any type of respiratory illness, and
especially epiglottitis, is suspected, putting any object, including a tongue
depressor for inspection or a cotton-tipped applicator to obtain a throat
culture, in the back of the mouth or throat or having the child open the mouth
is inappropriate because doing so may predispose the child to laryngospasm or
occlusion of the airway by a swollen epiglottis.
6. ANSWER C. The client is exhibiting the side effects associated with lithium
therapy that are temporary. Therefore, the nurse would continue the lithium
and explain to the client that he or she is experiencing temporary side effects
of lithium that will subside. Common side effects of lithium are nausea, dry
mouth, diarrhea, thirst, mild hand tremor, weight gain, bloating, insomnia,
and lightheadedness. Immediately notifying the physician about these common
side effects is not necessary.
7. ANSWER A. Thyroid replacement is a lifelong maintenance therapy. The
medication is usually given as one dose in the morning. It cannot be tapered or
discontinued, because the client needs thyroid supplementation to maintain
health. The medication cannot be discontinued after the TSH level is normal;
the dose will be maintained at the level that normalizes the TSH
concentration.
8. ANSWER D. A client who is beginning training for a tennis team would most
likely require an adjustment in lithium dosage because excessive sweating can
increase the serum lithium level, possibly leading to toxicity. Adjustments in
lithium dosage would also be necessary when other medications have been
added, when an illness with high fever occurs, and when a new diet begins.
9. ANSWER C. The most appropriate response is to continue all treatments and
attempt to stabilize the client using fluid replacement without administering
blood or blood products. It is imperative that the health care team respect the
client’s religious belief and wishes, even if they are not those of the health care
team. Discontinuing all measures is not an option. The health care team should
continue to provide the best care possible and does not need to notify the
attorney.
10. ANSWER B. Cystic fibrosis is the most common inherited disease in
children. It is inherited as an autosomal recessive trait, meaning that the child
inherits the defective gene from both parents. The chances are one in four for
each of this couple’s pregnancies.
11. ANSWER C. Nitroglycerin in all dosage forms (sublingual, transdermal, or
intravenous) should be shielded from light to prevent deterioration. Clients
should be instructed to keep the nitroglycerin in the dark container that is
supplied by the pharmacy, and it should not be removed or placed in any other
container.
12. ANSWER A, B, and D. Sinus bradycardia has the following characteristics:
1) P wave is normal and consistent in shape, occurring in front of every QRS
complex; 2) a ventricular and atrial rate less than 60 bpm; and 3) a PR
interval that is between 0.12 to 0.20 seconds. An atrial rate of 120 bpm
indicates tachycardia. ST segment elevation may indicate a myocardial
infarction.
13. ANSWER C. Valproic acid (Depakene) causes sedation as well as nausea,
vomiting, and indigestion. Sedation is important because the client needs to be
cautioned about driving or operating machinery that could be dangerous
while feeling sedated from the medication. Depakene does not cause an
increase in urination, slowed thinking, or weight loss. However, some clients
may experience weight gain.
14. ANSWER A. These effects and others when seen after birth are known as a
cluster of symptoms called fetal alcohol syndrome. Vitamin B6 and vitamin
Adeficiency can affect growth and development but not with these specific
effects. Folic acid deficiency contributes to neural tube defects.
15. ANSWER D. NSAIDs are irritating to the gastric mucosa and should be
taken with food. NSAIDs are usually taken once or twice daily. Joint exercise is
not related to the drug administration. Antacids may interfere with the
absorption of the drug.
16. ANSWER B. The client who drinks alcohol while taking disulfiram
(Antabuse) will experience sweating, flushing of the neck and face,
tachycardia, hypotension, a throbbing headache, nausea and vomiting,
palpitations, dyspnea, tremor, and/or weakness.
17. ANSWER C. Holding the gauze pledget against an intramuscular injection
site while removing the needle from the muscle avoids the discomfort of the
needle pulling on the skin.
18. ANSWER C. Physical activity is gradually increased after a myocardial
infarction while the client is still hospitalized and through a period of
rehabilitation. The client is progressing too rapidly if activity significantly
changes respirations, causing dyspnea, chest pain, a rapid heartbeat, or
fatigue. When any of these symptoms appears, the client should reduce activity
and progress more slowly. Edema suggests a circulatory problem that must be
addressed but doesn’t necessarily indicate overexertion. Cyanosis indicates
reduced oxygen-carrying capacity of red blood cells and indicates a severe
pathology. It is not appropriate to use cyanosis as an indicator for
overexertion. Weight loss is indicative of several factors but not overexertion.
19. ANSWER C. When a client talks about not having a problem with alcohol,
the nurse needs to point out how alcohol has gotten the client into trouble.
Concrete, factual information is helpful in decreasing the client’s denial that
alcohol is a problem. The other approaches allow the client to use defense
mechanisms, such as rationalization, projection, and minimization, to explain
her actions. Therefore, these approaches are not helpful.
20. ANSWER C. Medicaid is state funded, with matching federal funds, and
provides medical assistance for low-income persons without health insurance.
The program for older adults is Medicare.

Más contenido relacionado

La actualidad más candente

La actualidad más candente (20)

PEMBERIAN UBAT PARENTERAL
PEMBERIAN UBAT PARENTERALPEMBERIAN UBAT PARENTERAL
PEMBERIAN UBAT PARENTERAL
 
TRANFUSI DARAH
TRANFUSI DARAHTRANFUSI DARAH
TRANFUSI DARAH
 
CHEST TUBE & UNDER WATER SEAL
CHEST TUBE  &  UNDER WATER SEALCHEST TUBE  &  UNDER WATER SEAL
CHEST TUBE & UNDER WATER SEAL
 
respiratory DISTRESS SYNDROME
respiratory DISTRESS SYNDROME respiratory DISTRESS SYNDROME
respiratory DISTRESS SYNDROME
 
MENGAMBIL & INTERPRETASI E.C.G
MENGAMBIL & INTERPRETASI E.C.GMENGAMBIL & INTERPRETASI E.C.G
MENGAMBIL & INTERPRETASI E.C.G
 
INTUBASI
INTUBASIINTUBASI
INTUBASI
 
Konsep asepsis medikal dan sugikal
Konsep asepsis medikal dan sugikalKonsep asepsis medikal dan sugikal
Konsep asepsis medikal dan sugikal
 
BRONCHIAL ASTHMA
BRONCHIAL ASTHMABRONCHIAL ASTHMA
BRONCHIAL ASTHMA
 
1.9 bandaging copy
1.9 bandaging   copy1.9 bandaging   copy
1.9 bandaging copy
 
Jagaan bayi baru lahir
Jagaan bayi baru lahirJagaan bayi baru lahir
Jagaan bayi baru lahir
 
Acute gromerulonephritis
Acute gromerulonephritisAcute gromerulonephritis
Acute gromerulonephritis
 
Modul 9.5 Skill Kejururawatan
Modul 9.5   Skill KejururawatanModul 9.5   Skill Kejururawatan
Modul 9.5 Skill Kejururawatan
 
PENJAGAAN PLASTER OF PARIS dan PENJAGAAN TRAKSI
PENJAGAAN PLASTER OF PARIS dan PENJAGAAN TRAKSIPENJAGAAN PLASTER OF PARIS dan PENJAGAAN TRAKSI
PENJAGAAN PLASTER OF PARIS dan PENJAGAAN TRAKSI
 
DIABETES MELITUS
DIABETES MELITUSDIABETES MELITUS
DIABETES MELITUS
 
Pemberian Ubat Parenteral - Suntikan
Pemberian Ubat Parenteral - SuntikanPemberian Ubat Parenteral - Suntikan
Pemberian Ubat Parenteral - Suntikan
 
Keperluan eliminasi
Keperluan eliminasiKeperluan eliminasi
Keperluan eliminasi
 
Pneumotorak dan Hemotorak
Pneumotorak dan HemotorakPneumotorak dan Hemotorak
Pneumotorak dan Hemotorak
 
SISTEM ALIMENTARI - Penjagaan Kolostomi dan Irigasi Stoma
SISTEM ALIMENTARI - Penjagaan Kolostomi dan Irigasi StomaSISTEM ALIMENTARI - Penjagaan Kolostomi dan Irigasi Stoma
SISTEM ALIMENTARI - Penjagaan Kolostomi dan Irigasi Stoma
 
INCISION AND DRAINAGE
INCISION AND DRAINAGEINCISION AND DRAINAGE
INCISION AND DRAINAGE
 
PENJAGAAN PRA DAN POS SURGERY
PENJAGAAN PRA DAN POS SURGERYPENJAGAAN PRA DAN POS SURGERY
PENJAGAAN PRA DAN POS SURGERY
 

Similar a Contoh Soalan LJM

MCQ. Medical-surgical Nursing
MCQ. Medical-surgical NursingMCQ. Medical-surgical Nursing
MCQ. Medical-surgical NursingNursing Path
 
Fundamentals of nursing practice exam
Fundamentals of nursing practice examFundamentals of nursing practice exam
Fundamentals of nursing practice examNursing Path
 
300-Item NLE Practice Exam
300-Item NLE Practice Exam300-Item NLE Practice Exam
300-Item NLE Practice ExamScott Faria
 
PNLE Medical Surgical Nursing Exam 1.docx
PNLE Medical Surgical Nursing Exam 1.docxPNLE Medical Surgical Nursing Exam 1.docx
PNLE Medical Surgical Nursing Exam 1.docxAbdhaniePanontongan
 
Dr. RKJ~ Management of Seizure Disorders in Pregnengy~ in pregnency
Dr. RKJ~ Management of Seizure Disorders in Pregnengy~ in pregnencyDr. RKJ~ Management of Seizure Disorders in Pregnengy~ in pregnency
Dr. RKJ~ Management of Seizure Disorders in Pregnengy~ in pregnencyRahul Jaiswal
 
Fundamentals of nursing practice exam
Fundamentals of nursing practice examFundamentals of nursing practice exam
Fundamentals of nursing practice examNursing Path
 
Emergency nursing questionnaires
Emergency nursing questionnairesEmergency nursing questionnaires
Emergency nursing questionnairesJoan Delgado
 
council exams 2015- correction.docx
council exams 2015- correction.docxcouncil exams 2015- correction.docx
council exams 2015- correction.docxRBGroup
 
اسئلة تمريض 3
اسئلة تمريض 3اسئلة تمريض 3
اسئلة تمريض 3al asheery
 
Fundamentals of nursing practice exam 1
Fundamentals of nursing practice exam 1Fundamentals of nursing practice exam 1
Fundamentals of nursing practice exam 1Nursing Path
 
Nursing management of hemodialysis
Nursing management of hemodialysisNursing management of hemodialysis
Nursing management of hemodialysisMustafa Abdalla
 
EXAMEN COUNCIL QUESTION JUIN 2019.pdf
EXAMEN COUNCIL QUESTION JUIN 2019.pdfEXAMEN COUNCIL QUESTION JUIN 2019.pdf
EXAMEN COUNCIL QUESTION JUIN 2019.pdfRBGroup
 
Rwanda National council NCNM 2019 answer questions (1).pdf
Rwanda National council NCNM 2019 answer questions (1).pdfRwanda National council NCNM 2019 answer questions (1).pdf
Rwanda National council NCNM 2019 answer questions (1).pdfRBGroup
 
Fundamental of nursing quiz
Fundamental of nursing quizFundamental of nursing quiz
Fundamental of nursing quizNursing Path
 

Similar a Contoh Soalan LJM (20)

nclex
nclex nclex
nclex
 
MCQ. Medical-surgical Nursing
MCQ. Medical-surgical NursingMCQ. Medical-surgical Nursing
MCQ. Medical-surgical Nursing
 
1
11
1
 
Fundamentals of nursing practice exam
Fundamentals of nursing practice examFundamentals of nursing practice exam
Fundamentals of nursing practice exam
 
300-Item NLE Practice Exam
300-Item NLE Practice Exam300-Item NLE Practice Exam
300-Item NLE Practice Exam
 
Drill 8
Drill 8Drill 8
Drill 8
 
PNLE Medical Surgical Nursing Exam 1.docx
PNLE Medical Surgical Nursing Exam 1.docxPNLE Medical Surgical Nursing Exam 1.docx
PNLE Medical Surgical Nursing Exam 1.docx
 
Dr. RKJ~ Management of Seizure Disorders in Pregnengy~ in pregnency
Dr. RKJ~ Management of Seizure Disorders in Pregnengy~ in pregnencyDr. RKJ~ Management of Seizure Disorders in Pregnengy~ in pregnency
Dr. RKJ~ Management of Seizure Disorders in Pregnengy~ in pregnency
 
Fundamentals of nursing practice exam
Fundamentals of nursing practice examFundamentals of nursing practice exam
Fundamentals of nursing practice exam
 
6932118.ppt
6932118.ppt6932118.ppt
6932118.ppt
 
Emergency nursing questionnaires
Emergency nursing questionnairesEmergency nursing questionnaires
Emergency nursing questionnaires
 
council exams 2015- correction.docx
council exams 2015- correction.docxcouncil exams 2015- correction.docx
council exams 2015- correction.docx
 
اسئلة تمريض 3
اسئلة تمريض 3اسئلة تمريض 3
اسئلة تمريض 3
 
Fundamentals of nursing practice exam 1
Fundamentals of nursing practice exam 1Fundamentals of nursing practice exam 1
Fundamentals of nursing practice exam 1
 
Nursing management of hemodialysis
Nursing management of hemodialysisNursing management of hemodialysis
Nursing management of hemodialysis
 
Quiz for BDS, OMFS
Quiz for BDS, OMFSQuiz for BDS, OMFS
Quiz for BDS, OMFS
 
Drill 9
Drill 9Drill 9
Drill 9
 
EXAMEN COUNCIL QUESTION JUIN 2019.pdf
EXAMEN COUNCIL QUESTION JUIN 2019.pdfEXAMEN COUNCIL QUESTION JUIN 2019.pdf
EXAMEN COUNCIL QUESTION JUIN 2019.pdf
 
Rwanda National council NCNM 2019 answer questions (1).pdf
Rwanda National council NCNM 2019 answer questions (1).pdfRwanda National council NCNM 2019 answer questions (1).pdf
Rwanda National council NCNM 2019 answer questions (1).pdf
 
Fundamental of nursing quiz
Fundamental of nursing quizFundamental of nursing quiz
Fundamental of nursing quiz
 

Último

THEORIES OF ORGANIZATION-PUBLIC ADMINISTRATION
THEORIES OF ORGANIZATION-PUBLIC ADMINISTRATIONTHEORIES OF ORGANIZATION-PUBLIC ADMINISTRATION
THEORIES OF ORGANIZATION-PUBLIC ADMINISTRATIONHumphrey A Beña
 
Transaction Management in Database Management System
Transaction Management in Database Management SystemTransaction Management in Database Management System
Transaction Management in Database Management SystemChristalin Nelson
 
Active Learning Strategies (in short ALS).pdf
Active Learning Strategies (in short ALS).pdfActive Learning Strategies (in short ALS).pdf
Active Learning Strategies (in short ALS).pdfPatidar M
 
4.16.24 21st Century Movements for Black Lives.pptx
4.16.24 21st Century Movements for Black Lives.pptx4.16.24 21st Century Movements for Black Lives.pptx
4.16.24 21st Century Movements for Black Lives.pptxmary850239
 
Inclusivity Essentials_ Creating Accessible Websites for Nonprofits .pdf
Inclusivity Essentials_ Creating Accessible Websites for Nonprofits .pdfInclusivity Essentials_ Creating Accessible Websites for Nonprofits .pdf
Inclusivity Essentials_ Creating Accessible Websites for Nonprofits .pdfTechSoup
 
ICS2208 Lecture6 Notes for SL spaces.pdf
ICS2208 Lecture6 Notes for SL spaces.pdfICS2208 Lecture6 Notes for SL spaces.pdf
ICS2208 Lecture6 Notes for SL spaces.pdfVanessa Camilleri
 
ISYU TUNGKOL SA SEKSWLADIDA (ISSUE ABOUT SEXUALITY
ISYU TUNGKOL SA SEKSWLADIDA (ISSUE ABOUT SEXUALITYISYU TUNGKOL SA SEKSWLADIDA (ISSUE ABOUT SEXUALITY
ISYU TUNGKOL SA SEKSWLADIDA (ISSUE ABOUT SEXUALITYKayeClaireEstoconing
 
Grade 9 Quarter 4 Dll Grade 9 Quarter 4 DLL.pdf
Grade 9 Quarter 4 Dll Grade 9 Quarter 4 DLL.pdfGrade 9 Quarter 4 Dll Grade 9 Quarter 4 DLL.pdf
Grade 9 Quarter 4 Dll Grade 9 Quarter 4 DLL.pdfJemuel Francisco
 
AUDIENCE THEORY -CULTIVATION THEORY - GERBNER.pptx
AUDIENCE THEORY -CULTIVATION THEORY -  GERBNER.pptxAUDIENCE THEORY -CULTIVATION THEORY -  GERBNER.pptx
AUDIENCE THEORY -CULTIVATION THEORY - GERBNER.pptxiammrhaywood
 
How to Add Barcode on PDF Report in Odoo 17
How to Add Barcode on PDF Report in Odoo 17How to Add Barcode on PDF Report in Odoo 17
How to Add Barcode on PDF Report in Odoo 17Celine George
 
Student Profile Sample - We help schools to connect the data they have, with ...
Student Profile Sample - We help schools to connect the data they have, with ...Student Profile Sample - We help schools to connect the data they have, with ...
Student Profile Sample - We help schools to connect the data they have, with ...Seán Kennedy
 
MULTIDISCIPLINRY NATURE OF THE ENVIRONMENTAL STUDIES.pptx
MULTIDISCIPLINRY NATURE OF THE ENVIRONMENTAL STUDIES.pptxMULTIDISCIPLINRY NATURE OF THE ENVIRONMENTAL STUDIES.pptx
MULTIDISCIPLINRY NATURE OF THE ENVIRONMENTAL STUDIES.pptxAnupkumar Sharma
 
USPS® Forced Meter Migration - How to Know if Your Postage Meter Will Soon be...
USPS® Forced Meter Migration - How to Know if Your Postage Meter Will Soon be...USPS® Forced Meter Migration - How to Know if Your Postage Meter Will Soon be...
USPS® Forced Meter Migration - How to Know if Your Postage Meter Will Soon be...Postal Advocate Inc.
 
4.16.24 Poverty and Precarity--Desmond.pptx
4.16.24 Poverty and Precarity--Desmond.pptx4.16.24 Poverty and Precarity--Desmond.pptx
4.16.24 Poverty and Precarity--Desmond.pptxmary850239
 
What is Model Inheritance in Odoo 17 ERP
What is Model Inheritance in Odoo 17 ERPWhat is Model Inheritance in Odoo 17 ERP
What is Model Inheritance in Odoo 17 ERPCeline George
 
INTRODUCTION TO CATHOLIC CHRISTOLOGY.pptx
INTRODUCTION TO CATHOLIC CHRISTOLOGY.pptxINTRODUCTION TO CATHOLIC CHRISTOLOGY.pptx
INTRODUCTION TO CATHOLIC CHRISTOLOGY.pptxHumphrey A Beña
 
Field Attribute Index Feature in Odoo 17
Field Attribute Index Feature in Odoo 17Field Attribute Index Feature in Odoo 17
Field Attribute Index Feature in Odoo 17Celine George
 

Último (20)

THEORIES OF ORGANIZATION-PUBLIC ADMINISTRATION
THEORIES OF ORGANIZATION-PUBLIC ADMINISTRATIONTHEORIES OF ORGANIZATION-PUBLIC ADMINISTRATION
THEORIES OF ORGANIZATION-PUBLIC ADMINISTRATION
 
Transaction Management in Database Management System
Transaction Management in Database Management SystemTransaction Management in Database Management System
Transaction Management in Database Management System
 
Active Learning Strategies (in short ALS).pdf
Active Learning Strategies (in short ALS).pdfActive Learning Strategies (in short ALS).pdf
Active Learning Strategies (in short ALS).pdf
 
4.16.24 21st Century Movements for Black Lives.pptx
4.16.24 21st Century Movements for Black Lives.pptx4.16.24 21st Century Movements for Black Lives.pptx
4.16.24 21st Century Movements for Black Lives.pptx
 
Inclusivity Essentials_ Creating Accessible Websites for Nonprofits .pdf
Inclusivity Essentials_ Creating Accessible Websites for Nonprofits .pdfInclusivity Essentials_ Creating Accessible Websites for Nonprofits .pdf
Inclusivity Essentials_ Creating Accessible Websites for Nonprofits .pdf
 
ICS2208 Lecture6 Notes for SL spaces.pdf
ICS2208 Lecture6 Notes for SL spaces.pdfICS2208 Lecture6 Notes for SL spaces.pdf
ICS2208 Lecture6 Notes for SL spaces.pdf
 
ISYU TUNGKOL SA SEKSWLADIDA (ISSUE ABOUT SEXUALITY
ISYU TUNGKOL SA SEKSWLADIDA (ISSUE ABOUT SEXUALITYISYU TUNGKOL SA SEKSWLADIDA (ISSUE ABOUT SEXUALITY
ISYU TUNGKOL SA SEKSWLADIDA (ISSUE ABOUT SEXUALITY
 
Grade 9 Quarter 4 Dll Grade 9 Quarter 4 DLL.pdf
Grade 9 Quarter 4 Dll Grade 9 Quarter 4 DLL.pdfGrade 9 Quarter 4 Dll Grade 9 Quarter 4 DLL.pdf
Grade 9 Quarter 4 Dll Grade 9 Quarter 4 DLL.pdf
 
AUDIENCE THEORY -CULTIVATION THEORY - GERBNER.pptx
AUDIENCE THEORY -CULTIVATION THEORY -  GERBNER.pptxAUDIENCE THEORY -CULTIVATION THEORY -  GERBNER.pptx
AUDIENCE THEORY -CULTIVATION THEORY - GERBNER.pptx
 
How to Add Barcode on PDF Report in Odoo 17
How to Add Barcode on PDF Report in Odoo 17How to Add Barcode on PDF Report in Odoo 17
How to Add Barcode on PDF Report in Odoo 17
 
YOUVE_GOT_EMAIL_PRELIMS_EL_DORADO_2024.pptx
YOUVE_GOT_EMAIL_PRELIMS_EL_DORADO_2024.pptxYOUVE_GOT_EMAIL_PRELIMS_EL_DORADO_2024.pptx
YOUVE_GOT_EMAIL_PRELIMS_EL_DORADO_2024.pptx
 
Student Profile Sample - We help schools to connect the data they have, with ...
Student Profile Sample - We help schools to connect the data they have, with ...Student Profile Sample - We help schools to connect the data they have, with ...
Student Profile Sample - We help schools to connect the data they have, with ...
 
MULTIDISCIPLINRY NATURE OF THE ENVIRONMENTAL STUDIES.pptx
MULTIDISCIPLINRY NATURE OF THE ENVIRONMENTAL STUDIES.pptxMULTIDISCIPLINRY NATURE OF THE ENVIRONMENTAL STUDIES.pptx
MULTIDISCIPLINRY NATURE OF THE ENVIRONMENTAL STUDIES.pptx
 
USPS® Forced Meter Migration - How to Know if Your Postage Meter Will Soon be...
USPS® Forced Meter Migration - How to Know if Your Postage Meter Will Soon be...USPS® Forced Meter Migration - How to Know if Your Postage Meter Will Soon be...
USPS® Forced Meter Migration - How to Know if Your Postage Meter Will Soon be...
 
4.16.24 Poverty and Precarity--Desmond.pptx
4.16.24 Poverty and Precarity--Desmond.pptx4.16.24 Poverty and Precarity--Desmond.pptx
4.16.24 Poverty and Precarity--Desmond.pptx
 
FINALS_OF_LEFT_ON_C'N_EL_DORADO_2024.pptx
FINALS_OF_LEFT_ON_C'N_EL_DORADO_2024.pptxFINALS_OF_LEFT_ON_C'N_EL_DORADO_2024.pptx
FINALS_OF_LEFT_ON_C'N_EL_DORADO_2024.pptx
 
What is Model Inheritance in Odoo 17 ERP
What is Model Inheritance in Odoo 17 ERPWhat is Model Inheritance in Odoo 17 ERP
What is Model Inheritance in Odoo 17 ERP
 
INTRODUCTION TO CATHOLIC CHRISTOLOGY.pptx
INTRODUCTION TO CATHOLIC CHRISTOLOGY.pptxINTRODUCTION TO CATHOLIC CHRISTOLOGY.pptx
INTRODUCTION TO CATHOLIC CHRISTOLOGY.pptx
 
Field Attribute Index Feature in Odoo 17
Field Attribute Index Feature in Odoo 17Field Attribute Index Feature in Odoo 17
Field Attribute Index Feature in Odoo 17
 
LEFT_ON_C'N_ PRELIMS_EL_DORADO_2024.pptx
LEFT_ON_C'N_ PRELIMS_EL_DORADO_2024.pptxLEFT_ON_C'N_ PRELIMS_EL_DORADO_2024.pptx
LEFT_ON_C'N_ PRELIMS_EL_DORADO_2024.pptx
 

Contoh Soalan LJM

  • 1. Enam jam selepas dipasangkan pengaliarn under water seal, Puan Rama mengadu kesukaran bernafas. Tindakan segera anda ialah a. Lakukan sedutan nasofaringeal b. Nasihatkan pesakit menarik nafas panjang c. Mulakan terapi oksigen 4 liter seminit d. Periksa fluktasi paras air pengaliran under water seal 2. Intervensi kejururawatan yang dilakukan ke atas pesakit emphysema untuk mengelakkan small airway collapse adalah a. Melakukan postural drainage b. Melakukan chest percussion and vibration c. Galakkan pesakit melakukan pursed lip breathing d. Galakkan pesakit melakukan deep breathing exercise 3. Nasihat yang perlu diberikan kepada pesakit yang menghidap bronchitis bagi mencegah simptomnya menjadi lebih teruk adalah a. Lakukan senaman awal pagi b. Makan makanan yang tinggi fiber c. Elakkan dari tempat sesak dan berhabuk d. Hentikan antibiotic hanya apabila keadaan pulih 4. Cik Lim dipasang chest tube sebab mengalami haemotorax. Tujuan merawat beliau dalam posisi semi fowlers ialah untuk a. Mengelakkan pulmonary emboli b. Memastikan tiub penyaliran dada berfungsi dengan baik c. Menggalakkan penyaliran dan perkembangan paru-paru d. Membenarkan jururawat mencapai tiub dada dengan mudah
  • 2. 5. Chest tube dimasukkan pada Puan Salmi kerana mengalami pneumotorax. Chest tube diklamkan sebelum pengeluaran. Tiba-tiba beliau mengalami dispnea. Intervensi kejururawatan segera adalah a. Membuka klam b. Beri terapi oksigen c. Posisi semula pesakit d. Ajar pesakit senaman pernafasan 6. Encik Ravi telah dijadualkanuntuk menjalani torakotomi pada keesokkan hari. Tindakan kejururawatan untuk mengekalkan perkembangan paru-paru adalah a. Mengawal pengambilan cecair b. Mengekalkan rehat sepenuhnya c. Meletakkan klien pada posisi fowlers d. Mengajar klien senaman pernafasan 7. Encik Maniam mengalami chronic obstructive pulmonary disease. Intervensi kejururawatan untuk meningkatkan status nutrisi beliau adalah a. Mulakan hyperalimentation therapy b. Beri diet kalori tinggi mengikiut toleransi c. Meningkatkan paras oksigen sebelum makan d. Beri sedikit makanan berzat dengan kerap 8. Seorang klien dengan asthma mengalami wheezing. Jururawat memahami ini berlaku kerana a. Kurang pertukaran gas b. Mucus yang pekat dan likat c. Pengyekatan salur pernafasan d. Penyekatan aliran udara yang keluar dari paru-paru 9. Anda diarahkan memberi infuse Amilophyline 1gm dalam 500ml Dextrose 5% dalam tempoh masa 12 jam. Faktor titisan ialah 20. Berapakah kadar aliran infuse seminit yang perlu anda berikan?
  • 3. a. 5 b. 14 c. 83 d. 125 10. Tindakan bagi mengatasi masalah kejururawatan potensi kekurangan cecair berkaitan dengan hemoraj bagi klien menghidap dengan haemorragic fever ialah a. Berikan nutrisi yang sesuai mengikut jumlah kehilangan cecair b. Rawat klien dalam persekitaran yang selamat untuk mengelakkan kecederaan c. Beri cecair melalui tiub nasogastrik untuk menggantikan cecair yang hilang d. Lakukan gastric lavage dengan air sejuk untuk memberhentikan pendarahan 11. Puan Lim mendesak untuk pulang walaupun masih belum discaj oleh doctor. Apakah tindakan jururawat dalam situasi ini? a. Puan Lim perlu menandatangani boring AOR sebab beliau ingkar arahan doctor b. Jangan hiiiraukan permintaan Puan Lim dan haling beliau dddaripada meninggalkan hospital c. Terangkan kepada Puan Lim bahawa beliau dibenarkan pulang sebaik sahaja sembuh d. Beritahu Puan Lim bahawa beliau tidak akan diterima masuk semula ke hospital kerana ingkar arahan doctor 12. Encik Halim berumur 60 tahun mengalami cirit-birit yang teruk. Keputusan gas darah arteri (ABG) beliau adalah pH 7.25, HCO3 18mEq/L, PCO2 32mmHg, PO2 90mmHG dan BE -3. Keputusan beliau menunjukkan a. Alkalosis metabolic b. Alkalosis respiratori c. Asidosis metabolic d. Asidosis respiratori
  • 4. 13. Rosmah berumur 5 tahun diberi prednisolone untuk rawatan glomerulonefritis kronik. Dalam memantau kesan sampingan medikasi ini, jururawat perlu perhatikan untuk a. Anoreksia dan sembelit b. Tiada selera makan dan cirit birit c. Tanda dehidrasi dan susut berat badan d. Tanda retensi cecair dan penambahan berat badan 14. Cecair gentian pilihan untuk klien dewasa yang dimasukkan ke wad dengan renjatan hipovolumia adalah a. Mannitol b. Dextrose 10% c. Lactated Ringers d. Normal saline 0.45% 15. Yang manakah antara berikut intervensi kejururawatan yang diambil jika jumlah baki penyaliran dialysis peritoneum +1000ml? a. Lapor segera kepada doctor b. Panjangkan masa penyaliran keluar c. Ubah baringan pesakit dari sisi ke sisi d. Cepatkan kadar pengaliran masuk diasilat 16. Penyiasatan berikut adalah petunjuk yang unggul untuk menilai pengawalan diabetes seorang pesakit datang ke klinik susulan selepas 3 bulan? a. Fasting blood sugar b. Random blood sugar c. Analysis of microalbumin d. Glycosylated haemoglobin level 17. Linda berumur 10 tahun seorang penghidap Type 1 Diabetes merancang untuk mengikut kelas renang. Pendidikan pesakit yang perlu anda berikan ialah a. Tambahkan 10 unit regular insulin pada hari latihan
  • 5. b. Tangguhkan masa makan malam sehingga selepas kelas renang c. Mengawalatur dos insulin pada waktu pagi supaya peak action insulin mencapai pada masa renang d. Monitor paras glukos sebelum dan selepas berenang untuk mengubahsuai amaun makanan dan insulin yang diperlukan 18. Berikut adalah kenyataan yang benar dalam pendidikan pesakit kepada Encik Wong berusia 75 tahun dan kini menghidap Syndrome Inappropriate Anti Dieuretic Hormone (SIADH) a. Elakkan makan pisang dan oren b. Kurangkan pengambilan garam daLam diet c. Timbang dan rekod berat badan setiap hari d. Minum sekurang-kurangnya 1 ½ hingga 2 liter cecair sehari 19. Pemerhatian yang penting terhadap seorang pesakit 4 jam selepas tiroidektomi ialah a. Corak pernafasan b. Keserakan suara c. Tingling of hands d. Output urine 20. Puan Tina kembali ke wad selepas menjalani tiroidectomi dan pemerhatian dijalankan setiap jam. Anda akan segera memanggil doctor sekiranya Puan Tina mengalami a. Muntah dan cirit birit b. Kurang upaya menumpukan perhatian c. Tachycardia dan peningkatan tekanan darah d. Laryngeal stridor semasa inspirasi dan ekspirasi A: SOALAN OBJEKTIF Jawab semua soalan
  • 6. 1. Manakah tanda berikut menunjukkan angin ahmar? A. Sakit facial B. Pergerakan yang tidak stabil C. Pertukaran bau D. Hilang fungsi percakapan 2. Manakah berikut adalah manifestasi penyerangan iskemia transient? A .Mula dengfan cepat dan symptom meningkat dengan perlahan B. Symptom bermula dengan tiba-tiba dan dalam masa yang singkat C. Symptom bermula dengan cepat dan menyebabkan kehilangan sedar diri D. Symptom bermula dengan segera dan mengakibatkan kehilangan neurological. 3. Manakah intervensi berikut adalah kritikal dalam penjagaan klien selepas lumbar punktur? A. Puasakan klieMasukan kateter ‘indwelling fole B.Beri sedatif pos prosedur untuk menggalakan tidur C.Asses tempat punktur untuk pengeluaran cecair serebral spina artau pembentukan hematoma 4. Manakah ubat berikut diberi untuk mencegah kejadian angin ahmar?
  • 7. A Clonidine (Catapres) B Benazepril (Lotensin) C Methyldopa (Aldomet) D Dipyridamole (persantin) 5. Nisbah compression : breathing dalam one-rescuer cardiopulmonary resuscitation (CPR) ialah A 5 : 1 B 5 : 2 C15 : 1 D15 : 2 6. Jumlah darah yang dipam keluar oleh jantung dalam satu denyutan jantung dikenali sebagai Preload Afterload Stroke volume Cardiac output 7. Aritmia ialah satu keadaan gangguan kepada Isipadu strok
  • 8. Keluaran output System konduksi jantung Pengaliran darah ke arteri koronari 8. Hipertensi boleh mengakibatkan kegagalan jantung. Bahagian jantung yang manakah akan mengalami kegagalan dahulu? Atrium kiri Atrium kanan Ventrikel kiri Ventrikel kanan 9. Pesakit hipertensi dinasihat mengurangkan pengambilan garam dalam pemakanannya kerana garam akan Menambahkan cecair intraselular Menambahkan pengambilan vena Meningkatkan retensi cecair interstitial Menambahkan isipadu cecair intravascular 10.Pemerhatian yang perlu dilakukan ke atas pesakit yang mengalami cerebral vascular accident (CVA) adalah tanda
  • 9. A. Kegagalan ginjal B. Kongesi pulmonary C. Gangguan peredaran periferi D. Peningkatan tekanan intrakranium 11.Encik Samad dimasukkan ke wad dengan tekanan darah 170/110 mmHg. Tindakan kejururawatan pertama anda ialah A. Rehatkan pesalit di atas katil B. Lengkapkan data peribadi pesakit C. Mengambil sejarah perubatan D. Ambil tekanan darah serta merta 12.Mekanisma yang menyebabkan kesakitan dada dalam keadaan angina pectoris ialah A. tekanan darah yang tinggi B. Kesesakan di arteri pulmonary C. mengambil sejarah perubatan D. ambil tekanan darah serta merta
  • 10. 13.Encik John yang mengalami infarksi miokardium dua hari lepas, kini didapati demam low grade. Demam ini disebabkan oleh reaksi terhadap A. Nekrosis tisu B. Infeksi miokardium C. Infeksi nasokomial D. Bacterial endokarditis 14.Semasa pemberian ubat glyceryl trinitrate (GTN), arahan yang perlu diberi kepada pesakit ialah A. Telan ubat bersama segelas air suam B. Menguyah ubat tersebut sebelum menelan C. Biarkan ubat itu larut perlahan-lahan di bawah lidah D. Ambil makanan selepas setengah jam menelan ubat 15.Tujuan utama pemerhatian rapi terhadap pesakit acute myocardial infarction yang sedang menerima infuse streptokinase adalah untuk mengesan A. Takipnea B. Pendarahan C. Fluid overload D. Extension of infarction
  • 11. 16.Komplikasi utama yang menyebabkan kadar mortality yang tinggi myocardial infarction adalah disebabkan oleh A. Dysrhytmias B. Pulmonary edema C. Left ventricle failure D. Aortic aneurysm 17.Edema yang dialami oleh pesakit congestive cardiac failureadalah disebabkan oleh A Peningkatan tekanan vena B Kekurangan serum albumin C Ketelapan membarn sel-sel D Kehilangan fungsi injap vena 18.Tindakan perawatan kepada pesakit kegagalan jantung yang mengalami edema di kaki A. Pakaikan supportive stockinet di kaki
  • 12. B. Beri sokongan pada kaki semasa mengubah baringan C. Galakkan pesakit melakukan senaman isometric kaki D. Tinggikan kaki pesakit ke paras jantung semasa rehat 19.Tujuan utama merawat pesakit yang mengalami congestive cardiac failure dalam posisi Fowler’s ialah A. Meningkatkan preload B. Meningkatkan tensi oksigen C. Mengurangkan retensi cecair D. Mengurangkan venous return 20.Pesakit kegagalan jantung kongestif mengeluarkan kahak yang berbuih dan berdarah akibat A. Infeksi di paru-paru B. Kongesi kapilari pulmonary C. Kesempitan salur pernafasan D. Penyekatan salur pernafasan 21.Kegagalan penutupan foramen ovale selepas bayi dilahirkan akan mengakibatkan
  • 13. A. Tetralogy of Fallot B. Atrial septal defect C. Pulmonary stenosis D. Patent ductus arteriosus 22.Tanda yang ditunjukkan oleh pesakit yang mengalami gangguan vascular arteri periferi ialah A. Edema di kedua-dua belah kaki B. Kadar nadi menurun di ekstremiti C. Kongesi vena di bahagian distal kaki D. Sakit kejang pada otot-otot betis dan kaki 23.May adalah seorang penghidap infektif endokarditis. Beliau berisiko mengalami cerebrovascular accident akibat A. Iskemia B. Aneurysm C. Embolism D. Pendarahan
  • 14. 24.Tab. Captopril yang dipreskripsikan kepada pesakit hypertensi boleh menurunkan tekanan darah dengan A. Mengurangkan isipadu intravascular B. Menyekat beta-receptor di salur darah periferi C. Mengakibatkan relaksasi otot-otot licin vascular D. Mengurangkan pertukaran angiotensin I ke angiotensin II 25.Tujuan suntikan morphine diberi kepada pesakit myocardial infarction ialah A. Mengelakan disritma jantung B. Menguatkan kotraktiliti jantung C. Mengurangkan kesakitan dan kegelisahan D. Meningkatkan pengaliran darah ke myocardium 26.Pesakit yang mengalami ‘pernicious anemia’ selepas pembedahan gastrectomy adalah disebabkan kekurangan A. Zat besi B. Asid folik
  • 15. C. Vitamin B12 D. Asid hidroklorik 27.Angina pectoris berlaku disebabkan oleh A. Nekrosis tisu miokardium B. Kekurangan oksigen miokardium C. Kekurangan keluaran kardiak D. Kelemahan kotraktiliti kardiak 28.Disritmia bermaksud keabnormalan A. Struktur jantung B. Perfusi jantung C. Kontraktiliti jantung D. Fungsi injap jantung 29.Demam rheumatic akut boleh mengakibatkan kerosakan pada A. Septum jantung B. Miokardium
  • 16. C. Endokardium D. Injap jantung 30.Tujuan intervensi bagi deep vein thrombosis ialah untuk A. Mencegah pendarahan B. Mengelak kesekatan vena C. Mengurangkan kejadian iskemia D. Mengelak embolisme pulmonari 31.Tujuan pemberian streptokinase pada pesakit yang mengalami infeksi miokardium A. Mencegah pembekuan darah B. Menambah kotraktiliti jantung C. Menghancur bekuan darah D. Mengawal arrhythmia 32.Selepas serangan infarksi miokardium, suhu badan pesakit meningkat dalam masa 24-48 jam kerana
  • 17. A. Reaksi terhadap tisu nekrosis B. Sakit dada yang teruk C. Mendapa endokarditis bacteria D. Mendapat infeksi nosokomial 33.Sebelum pemberian digoxin kepada pesakit, jururawat perlu menilai A. Denyutan apeks B. Kadar nadi C. Tekanan darah D. Kadar pernafasan 34.Pemberian cecair berlebihan melalui infuse intravena bagi pesakit yang mengalami kegagalan jantung kongestif boleh mengakibatkan A. Ascitis B. Hiperkalemia C. Hipernatremia D. Edema pulmonari 35.Keadaan ini sering dikaitkan dengan masalah arteri koronari A. Polisitemia
  • 18. B. Arteriosklerosis C. Embolisme D. Anemia 36.Pesakit yang mengalami kegagalan jantung kongestif mengeluarkan kahak yang bercampur darah kerana A. Jangkitan paru-paru B. Pendarahan gusi C. Kecederaan kapilari D. Hemoptesis 37.Pesakit yang mengalami endokarditis boleh mendapatkan strok kerana A. Pendarahan B. Embolisme C. Iskemia D. Aneurisme
  • 19. 38.Berikut adalah nasihat yang anda beri kepada seorang pesakit yang mengalami hipertensi A. Makan ubat mengikut masa dan kurangkan dos bila tekanan darah menurun B. Makan makanan seimbang mengikut kegemaran sendiri C. Mengenalpasti dan elak punca stress D. Lakukan aktiviti harian tanpa had 39.Pesakit yang menghidapi jaundis obstruktif mudah berdarah kerana kekurangan bahan ini A. Bilirubin B. Vitamin B12 C. Vitamin K D. Globulin 40.Posisi yang sesuai bagi pesakit yang mengalami CVA ( cerebral vascular accident) dengan tekanan intrakranium yang tinggi ialah A. Rekumben
  • 20. B. Pron C. Tinggikan bahagian kaki D. Tinggikan bahagian kepala 41.Tindakan perawatan serta merta terhadap pesakit yang mengalami tekanan darah tinggi 170/110 mmHg semasa kemasukan wad A. Beri ubat hipertensi B. Beri sokongan emosi C. Rehatkan pesakit D. Laporkan kepada doctor 42.Manifestasi klinikal spesifik infektif endokarditis A. Eritma B. Petekia C. Perdarahan gusi D. Murmur
  • 21. 43.Bekuan darah yang menyekat pengaliran vena dikenali sebagai A. Varicose vein B. Thrombosis vena C. Thrombophlebitis D. Embolism 44.Kanak-kanak yang menghidapi ‘Tetralogy of Falot’ akan menunjukkan satu ciri klinikal berikut A. Batuk B. Edema C. Sinosis D. Demam 45.Tujuan rehat penting bagi pesakit yang mengalami kegagalan jantung kongestif untuk A. Mengelakan edema B. Menggalakan selera makan
  • 22. C. Mengurangkan bebanan ke atas jantung D. Menggalakan kontraktiliti jantung 46.Bunyi pertama ‘lubb’ yang didengar semasa auscultation dengan stethoscope dihasilkan oleh penutupan A. Injap mitral B. Injap aorta C. Injap-injap semilunar D. Injap-injap atrioventrikular 47.Antara berikut yang manakah dua (2) vena utama yang bercantum membentuk vena portal hepatik? A. Vena mesentrik superior dan vena splenik B. Vena splenik dan vena gastroepiploic kanan C. Vena mesentrik superior dan mesentrik inferior D. Vena mesentrik inferior dan vena gastroepiploic kanan 48.Duktus limfatik kanan dan duktus torasik mengalir secara lansung ke A. Cistern chili B. Subclavian veins
  • 23. C. Superior vena cava D. Axillary lymph nodes 49.Semasa infeksi di system saraf, bilangan sel yang akan meningkat ialah A. Astrosit B. Microglia C. Ependima D. Oligodendrosit 50.Tindakan ubat Captopril (capoten) yang dipreskripsikan kepada pesakit hipertensi ialah untuk A. Menghalang rembesan aldosteron B. Merelakskan otot licin salur darah C. Menyekat penyerapan semula sodium D. Mengurangkan spasma ke otot licin salur darah 51.Anemia boleh berlaku selepas gastrektomi separa kerana
  • 24. A. Kekurangan factor intrinsik B. Saiz gaster sudah menjadi kecil C. Penghasilan jus gaster berkurangan D. Kelambatan pencernaan zat makanan 52.Ubat untuk rawatan angina A. Morphine B. Betaloc C. Streptokinase D. Glycerin trinitrate 53.Berikut adalah komplikasi infarksi miokardium KECUALI A. Disritmia B. Edema pulmonari C. Kegagalan kardiak D. Peningkatan keluaran kardiak 54.Kesan hipertensi kronik ke atas jantung adalah
  • 25. A. Stenosis injap mitral B. Hipertrofi myocardium C. Penurunan keluaran kahak D. Peningkatan keluaran kahak 55.Gangguan dalam konduksi jantung akan mengakibatkan A. Aritmia B. Hipotensi C. Ateriosklerosis D. Edema pulmonari 56.Denyutan dalam sistem konduksi jantung akan mengakibatkan A. Fluter atrium B. Ritma sinus C. Aritmia sinus D. Fibrilasi atrium 57.Kesan kegagalan jantung kanan ialah
  • 26. A. Angina pectoris B. Edema sistemik C. Edema pulmonari D. Infarksi miokardium 58.Langkah pertama dalam resusitasi kardiopulmonari ialah A. Memeriksa nadi carotid B. Membuka saluran C. Memulih denyutan jantung D. Memulakan kompresi jantung 59.Kompresi jantung hanya dilakukan keatas jantung mangsa selepas menentukan ketiadaan nadi A. Radial B. Brakial C. Carotid D. Temporal
  • 27. 60.Tujuan pemberian makanan lembut kepada pesakit infarksi miokardium akut adalah untuk A. Mengelakkan muntah B. Merehatkan sistem pencernaan C. Mengurangkan beban jantung D. Mengelakkan sembelit Part 3 1. The nurse is preparing a teaching plan for a 45-year-old client recently diagnosed with type 2 diabetes mellitus. What is the first step in this process? a. Establish goals. b. Choose video materials and brochures. c. Assess the client’s learning needs. d. Set priorities of learning needs. 2. A loading dose of digoxin (Lanoxin) is given to a client newly diagnosed with atrial fibrillation. The nurse begins instructing the client about the medication and the importance of monitoring the heart rate. An expected outcome of the education program will be: a. a return demonstration of palpating the radial pulse. b. a return demonstration of how to take the medication. c. verbalization of why the client has atrial fibrillation. d. verbalization of the need for the medication. 3. A multigravida client is scheduled for a percutaneous umbilical blood sampling (PUBS) procedure. The nurse instructs the client that this procedure is useful for diagnosing which of the following? a. Twin pregnancies. b. Fetal lung maturation. c. Rh disease. d. Alpha-Fetoprotein level.
  • 28. 4. Which of the following is a side effect of vancomycin (Vancocin) and needs to be reported promptly? a. Vertigo. b. Tinnitus. c. Muscle stiffness. d. Ataxia. 5. Which of the following statements indicates that the client with a peptic ulcer understands the dietary modifications he will need to follow at home? a. “I should eat a bland, soft diet.” b. “It is important to eat six small meals a day.” c. “I should drink several glasses of milk a day.” d. “I should avoid alcohol and caffeine.” 6. The client with a nasogastric tube begins to complain of abdominal distention. Which of the following measures should the nurse implement first? a. Call the physician. b. Irrigate the nasogastric tube. c. Check the function of the suction equipment. d. Reposition the nasogastric tube. 7. A male client has been diagnosed as having a low sperm count during infertility studies. After instructions by the nurse about some causes of low sperm counts, the nurse determines that the client needs further instructionswhen he says low sperm counts may be caused by which of the following? a. Varicocele. b. Frequent use of saunas. c. Endocrine imbalances. d. Decreased body temperature. 8. The nurse assesses a client and notes puffy eyelids, swollen ankles, and crackles at both lung bases. The nurse understands that these clinical findings are most specifically associated with fluid excess in which of the following compartments? a. Interstitial compartment. b. Intravascular compartment. c. Extracellular compartment. d. Intracellular compartment. 9. An expected physiologic response to a low potassium level is
  • 29. a. cardiac dysrhythmias. b. hyperglycemia. c. hypertension. d. increased energy. 10. When teaching unlicensed assistive personnel (UAP) about the importance of handwashing in preventing disease, the nurse makes which of the following statements? a. “It is not necessary to wash your hands as long as you use gloves.” b. “Handwashing is the best method for preventing cross-contamination.” c. “Waterless commercial products are not effective for killing organisms.” d. “The hands do not serve as a source of infection.” 11. The nurse is performing Leopold maneuvers on a woman who is in her eighth month of pregnancy. The nurse is palpating the uterus at the symphysis pubis area. Which of the following maneuvers is the nurse performing? a. First maneuver. b. Second maneuver. c. Third maneuver. d. Fourth maneuver. 12. A client in a cardiac rehabilitation program states that he would like to make sure he is eating the right foods to ensure adequate endurance on the treadmill. Which of the following nutrients is most helpful for promoting endurance during sustained activity? a. Protein. b. Carbohydrate. c. Fat. d. Water 13. A client’s chest tube is connected to a chest tube drainage system with a water seal. The nurse notes that the fluid in the water-seal column is fluctuating with each breath that the client takes. The fluctuation means that a. there is an obstruction in the chest tube. b. the client is developing subcutaneous emphysema. c. the chest tube system is functioning properly. d. there is a leak in the chest tube system. 14. A client with diabetes is explaining to the nurse how she will care for her feet at home. Which statement indicates that the client understands proper foot care? a. “When I injure my toe, I will plan to put iodine on it.” b. “I should inspect my feet at least once a week.”
  • 30. c. “I do not plan to wear shoes while I am in the house.” d. “It is important to dry my feet carefully after my bath.” 15. The nurse assesses a client with diverticulitis and suspects peritonitis when which of the following symptoms is noted? a. Hyperactive bowel sounds. b. Rigid abdominal wall. c. Explosive diarrhea. d. Excessive flatulence. 16. When assessing a client, which risk factors would lead the nurse to suspect that the client has pancreatitis? Select all that apply. a. Excessive alcohol use. b. Gallstones. c. Abdominal trauma. d. Hyperlipidemia with excessive triglycerides. 17. When performing chest percussion on a child, which of the following techniques would the nurse use? a. Firmly but gently striking the chest wall to make a popping sound. b. Gently striking the chest wall to make a slapping sound c. Percussing over an area from the umbilicus to the clavicle. d. Placing a blanket between the nurse’s hand and the child’s chest. 18. The nurse walks into the room of a client who has a “Do Not Resuscitate” order and finds the client without a pulse, respirations, or blood pressure. What is the most appropriate action? a. Stay in the room and notify the nursing team for assistance. b. Push the emergency alarm to call a code. c. Dial the hospital phone number for a code. d. Pull the curtain and leave the room. 19. A client is trying to lose weight at a moderate pace. If the client eliminates 1000 calories per day from his normal intake, how many pounds would he lose in 1 week? a. 1 pound. b. 2 pounds. c. 3 pounds. d. 4 pounds. 20. A nulliparous client calls the clinic and tells the nurse that she forgot to take her oral contraceptive this morning. Which of the following would the nurse instruct the client to do?
  • 31. a. Take the medication immediately. b. Restart the medication in the morning. c. Use another form of contraception for 2 weeks. d. Take two pills tonight before bedtime.
  • 32. Part 1 1. A client with HIV and AIDS confides that he is homosexual and his employer does not know his HIV status. The nurse’s best response to him is: A. “Would you like me to help you tell them?” B. “The information you confide in me is confidential.” C. “I must share this information with your family.” D. “I must share this information with your employer.” 2. The mother of a child with bronchial asthma tells the nurse that the child wants a pet. Which of the following pets should the nurse tell the mother is most appropriate? A. Cat. B. Fish. C. Gerbil. D. Canary. 3. An elderly client is being admitted to same-day surgery for cataract extraction. The client has several diamond rings. The nurse should explain to the client that A. her rings will be taped before the surgery. B. she will sign a valuables envelope that will be placed in a safe. C. the rings will be locked in the narcotics box. D. the nursing supervisor will hold onto the rings during the surgery. 4. When an infant resumes taking oral feedings after surgery to correct intussusception, the parents comment that the child seems to suck on the pacifier more since the surgery. The nurse explains that sucking on a pacifier A. provides an outlet for emotional tension. B. indicates readiness to take solid foods. C. indicates intestinal motility. D. is an attempt to get attention from the parents. 5. A 22-year-old client is brought to the emergency department with his fiancée after being involved in a serious motor vehicle accident. His GlasgowComa Scale score is 7 and he demonstrates evidence of decorticate posturing. Which of the following would be appropriate for obtaining permission to place a catheter for intracranial pressure (ICP) monitoring?
  • 33. A. The nurse will obtain a signed consent from the client’s fiancée because he is of legal age and they are engaged to be married. B. The physician will get a consultation from one other physician and proceed with placement of the ICP catheter until the family arrives to sign the consent. C. Two nurses will receive a verbal consent by telephone from the client’s next of kin before inserting the catheter. D. The physician will document the emergency nature of the client’s condition and that an ICP catheter for monitoring was placed without a consent. 6. A 68-year-old client’s daughter is asking about the follow-up evaluation for her father after his pneumonectomy for primary lung cancer. The nurse’s best response is which of the following? A. “The usual follow-up is chest x-ray and liver function tests every 3 months.” B. “The follow-up for your father will be a chest x-ray and a computed tomography (CT) scan of the abdomen every year.” C. “No follow-up is needed at this time.” D. “The follow-up for your father will be a chest x-ray every 6 months.” 7. The nurse is preparing to administer blood to an otherwise healthy client who requires postoperative blood replacement. The nurse is aware that the blood administration set must include A. a micron mesh filter. B. a nonfiltered administration blood set. C. a special leukocyte-poor filter. D. a microdrip administration set. 8. Under which circumstance may a nurse communicate medical information without the client’s consent? A. When certifying the client’s absence from work. B. When requested by the client’s family. C. When treating clients who have a sexually transmitted disease (STD). D. When ordered by another physician. 9. During the health history interview, which of the following strategies is the most effective for the nurse to use to help clients feel that they have an active role in their health care? A. Ask clients to complete a questionnaire. B. Provide clients with written instructions. C. Ask clients for their description of events and for their views concerning past medical care. D. Ask clients if they have any questions.
  • 34. 10. A client with severe major depression states, “My heart has stopped and my blood is black ash.” The nurse interprets this statement to be evidence of which of the following? A. Hallucination. B. Illusion. C. Delusion. D. Paranoia. 11. When a client wants to read his chart, the nurse should A. call the doctor to obtain permission. B. give the client the chart and answer questions for him. C. tell the client that he can read the chart when the doctor makes rounds. D. ask the client what he wants to know and answer those questions without giving him the chart. 12. A client who has a fractured leg has been instructed to ambulate without weight bearing on the affected leg. The nurse evaluates that the client is ambulating correctly if she uses which of the following crutch-walking gaits? A. Two-point gait. B. Four-point gait. C. Three-point gait. D. Swing-to gait. 13. A client with major depression states, “Life isn’t worth living anymore. Nothing matters.” Which of the following responses by the nurse would be best? A. “Are you thinking about killing yourself?” B. “Things will get better, you know.” C. “Why do you think that way?” D. “You shouldn’t feel that way.” 14. A client with bipolar 1 disorder has been prescribed olanzapine (Zyprexa) 5 mg two times a day and lamotrigine (Lamictal) 25 mg two times a day. Which of the following adverse effects would the nurse report to the physician immediately? Select all that apply. A. Rash. B. Nausea. C. Hyperthemia. D. Muscle rigidity. 15. A client is prescribed atropine, 0.4 mg IM. The atropine vial is labeled 0.5 mg/mL. How many milliliters should the nurse plan to administer?
  • 35. A. 0.8. B. 0.4 C. 8 D. 0.5 16. A multiparous client tells the nurse that she is using medroxyprogesterone (Depo-Provera) for contraception. The nurse instructs the client to increase her intake of which of the following? A. Folic acid. B. Vitamin C. C. Magnesium D. Calcium. 17. Which of the following statements made by a woman in the first trimester are consistent with this stage of pregnancy? Select all that apply. A. “My husband told his friends we will have to give up the mustang for a minivan.” B. “Oh my, how did this happen? I don’t need this now.” C. “I can’t wait to see my baby. Do you think it will have my blond hair and blue eyes?” D. “I wonder how it will feel to buy maternity clothes and be fat.” 18. The nurse is teaching a client about topical gentamicin sulfate (Garamycin). Which of the following comments by the client indicates the need for additional teaching? A. “I will avoid being out in the sun for long periods.” B. “I should stop applying it once the infected area heals.” C. “I’ll call the physician if the condition worsens.” D. “I should apply it to large open areas.” 19. A client has been taking imipramine (Tofranil) for his depression for 2 days. His sister asks the nurse, “Why is he still so depressed?” Which of the following responses by the nurse would be most appropriate? A. “Your brother is experiencing a very serious depression.” B. “I’ll be sure to convey your concern to his physician.” C.”It takes 2 to 4 weeks for the drug to reach its full effect.” D. “Perhaps we’ll need to change his medication.” 20. Which interventions would the nurse use to assist the client with grandiose delusions? Select all that apply.
  • 36. A. Accepting the client while not arguing with the delusion. B. Focusing on the feelings or meaning of the delusion. C. Focusing on events and topics based in reality. D. Confronting the client’s beliefs. NCLEX Comprehensive Exam Part 1 Questions Answer B. The nurse is responsible for maintaining confidentiality of this disclosure by the client. Answer B. Pets are discouraged when parents are trying to allergy-proof a home for a child with bronchial asthma, unless the pets are kept outside. Pets with hair or feathers are especially likely to trigger asthma attacks. A fish would be a satisfactory pet for this child, but the parents should be taught to keep the fish tank clean to prevent it from harboring mold Answer B. Under the policy for valuables, the nurse documents the description on an envelope with the client, the client and nurse sign the envelope, and the valuables envelope is locked in the safe. The other options increase the risk of loss or damage to the client’s valuables. Answer A. Sucking provides the infant with a sense of security and comfort. It also is an outlet for releasing tension. The infant should not be discouraged from sucking on the pacifier. Fussiness and irritability after feeding may indicate that the infant’s appetite is not satisfied. Sucking is not manipulative in the sense of seeking parental attention. Answer D. In a life-threatening emergency where time is of the essence in saving life or limb, a consent is not required. This client has a Glasgow Coma Scale score of 7, which means he is comatose. The client has deteriorated to a level where he cannot be aroused; withdraws in a purposeless manner from painful stimuli; exhibits decorticate posturing; and may or may not have brain stem reflexes intact. The placement of the ICP monitor is crucial to determine cerebral blood flow and prevent herniation. The client’s fiancée can not sign his consent because, until she is his wife or has designated power of attorney, she is not considered his next of kin. The physician should insert the catheter in this emergency. He does not need to get a consultation from one other physician. When a consent is needed for a situation that is not a true emergency, two nurses can receive a verbal consent by telephone from the client’s next of kin.
  • 37. Answer D. Follow-up generally involves semiannual chest radiographs. Recurrence usually occurs locally in the lungs and may be identified on chest radiographs. Follow-up after cancer treatment is an important component of the treatment plan. Serum markers (eg, liver function tests) have not been shown to detect recurrence of lung cancer. There are no data to support the need for an abdominal CT scan. Answer A. All blood products should be administered through a micron mesh filter. Blood is never administered without a filter. Leukocytes can be removed by using leukocyte-poor filters, and this is recommended to decrease reactions in clients such as hemophiliacs who require frequent transfusions. Blood is too concentrated to administer through a microdrip set. Answer C. STDs are communicable diseases that must be reported. The nurse is responsible for reporting these diseases to the appropriate public health agency, and to otherwise maintain the client’s confidentiality. The client’s family cannot request release of medical information without the client’s consent. A physician’s order is not a substitute for a client’s consent to release medical information in the absence of a communicable disease. Answer C. One of the best strategies to help clients feel in control is to ask them their view of situations, and to respond to what they say. This technique acknowledges that clients’ opinions have value and relevance to the interview. It also promotes an active role for clients in the process. Use of a questionnaire or written instructions is a means of obtaining information but promotes a passiveclient role. Asking whether the client has questions encourages client participation, but alone it does not acknowledge the client’s views. Answer C. A client with severe depression may experience symptoms of psychosis such as hallucinations and delusions that are typically mood congruent. The statement, “My heart has stopped and my blood is black ash,” is a mood-congruent somatic delusion. A delusion is a firm, false, fixed belief that is resistant to reason or fact. A hallucination is a false sensory perception unrelated to external stimuli. An illusion is a misinterpretation of a real sensory stimulus. Paranoia refers to suspiciousness of others and their actions. Answer B. The client should be allowed to see his chart. As a client advocate, the nurse should answer questions for the client. The nurse helps the client understand that he is a primary partner in the health team. The Bill of Rights for Patients has existed since the 1960s, and every client should be aware of this document. The doctor should not need to give permission for the client to see his chart. As a client advocate, the nurse should not make excuses to put the client off in regard to seeing his chart.
  • 38. Answer C. The three-point gait, in which the client advances the crutches and the affected leg at the same time while weight is supported on the unaffected extremity, is the appropriate gait of choice. This allows for non–weight bearing on the affected extremity. The two-point, four-point, and swing-to gaits require some weight bearing on both legs, which is contraindicated for this client. Answer A. When the client verbalizes that life isn’t worth living anymore, the nurse needs to ask the client directly about suicide by saying, “Are you thinking about killing yourself?” Asking directly does not provoke suicide but conveys concern, understanding, and the worth of the client. Often, the client experiences a sense of relief that someone finally hears him. It also helps the nurse plan responsible care by identifying the client who is at risk for suicide. The nurse would then evaluate the seriousness of the suicidal ideation by inquiring about the intent and plan. Stating, “Things will get better,” offers hope too soon without first evaluating the intent of the suicidal ideation. Asking, “Why do you think that way,” implies a lack of understanding and knowledge on the part of the nurse. Major depression usually is endogenous and biochemically based. Therefore, the client may not know why he doesn’t want to live. Saying, “You shouldn’t feel that way,” admonishes the client, decreases self-worth, and conveys a lack of understanding. Answer A, C and D. Lamotrigine (Lamictal), an antiepileptic, is used as a mood stabilizer for clients with bipolar disorder and has been found to be effective for the depressive phase of bipolar disorder. Common adverse effects are dizziness, headache, sedation, tremor, nausea, vomiting, and ataxia. The development of a rash needs to be reported and evaluated by the physician because it could indicate the start of a severe systemic rash known as Stevens- Johnson syndrome, a toxic epidermal necrolysis, which would necessitate the discontinuance of lamotrigine. Hyperthermia in conjunction with muscle rigidity suggests the development of the neuroleptic malignant syndrome, a life-threatening complication associated with olanzapine. Answer A. 0.4 mg/x = 0.5 mg/1 mL 0.4 = 0.5x 0.4/0.5 = x 0.8 ml = x. Answer D. The nurse should instruct the client to increase her intake of calcium because there is a slight increase in the risk of osteoporosis with this medication. Weight-bearing exercises are also advised. The drug may also impair glucose tolerance in women who are at risk for diabetes. Answer A, B and D. The first trimester is when the couple works through the psychological task of accepting the pregnancy. These statements describe the client and her partner coping with the pregnancy, how it feels, and how it will impact their lives. The feelings include pleasure, excitement and ambivalence.
  • 39. Wondering what the baby will look like and planning for the baby’s room occur later in the pregnancy. Answer D. The aminoglycoside antibiotic gentamicin sulfate (Garamycin) should not be applied to large denuded areas because toxicity and systemic absorption are possible. The nurse should instruct the client to avoid excessive sun exposure because gentamicin sulfate can cause photosensitivity. The client should be instructed to apply the antibiotic cream or ointment for only the length of time prescribed, because a superinfection can occur from overuse. The client should contact the physician if the condition worsens after use. Answer C. The nurse needs to inform the sister that there is a lag time of 2 to 4 weeks before a full clinical effect occurs with the drug. The nurse should let her know that her brother will gradually get better and symptoms of depression will improve. Telling the sister that her brother is experiencing a very serious depression does not give the sister important information about the medication. Additionally, this statement may cause alarm and anxiety. Conveying the sister’s concern to the physician does not provide her with the necessary information about the client’s medication. Telling the sister that the client’s medication may need to be changed is inappropriate because a full clinical effect occurs after 2 to 4 weeks. Answer C. For the client with grandiose delusions, the nurse would accept the client but not argue with the delusion to build trust and the client’s self- esteem. Focusing on the underlying feeling or meaning of the delusion helps to meet the client’s needs. Focusing on events and topics based in reality distracts the client from the delusional thinking. Confronting the client’s delusions or beliefs can lead to agitation in the client and the need to cling to the grandiose delusion to preserve self-esteem. Interacting with the client only when he is based in reality ignores the client’s needs and therapeutic nursing intervention. Part 2 1. A multigravida client visiting the prenatal clinic at 16 weeks’ gestation exhibits facial swelling, a brownish vaginal discharge, and fundal height of 22 cm. The client’s blood pressure is 160/90 mm Hg and her pulse is 80 bpm. The nurse interprets these findings as suggestive of which of the following? a. Placenta previa. b. Fetal anemia. c. Multifetal pregnancy. d. Gestational trophoblastic disease.
  • 40. 2. Which of the following responses would be most helpful for a client who is euphoric, intrusive, and interrupts other clients engaged in conversations to the point where they get up and leave or walk away? a. “When you interrupt others, they leave the area.” b. You are being rude and uncaring.” c. “You should remember to use your manners.” d. “You know better than to interrupt someone.” 3. The nurse coordinates with the laboratory staff to have the gentamicin trough serum level drawn. At what time should the blood be drawn in relation to the administration of the intravenous dose of gentamicin sulfate (Garamycin)? a. 2 hours before the administration of the next intravenous dose. b. 3 hours before the administration of the next intravenous dose. c. 4 hours before the administration of the next intravenous dose. d. Just before the administration of the next intravenous dose. 4. Older adults with known cardiovascular disease must balance which of the following measures for optimum health? a. Diet, exercise, and medication. b. Stress, hypertension, and pain. c. Mental health, diet, and stress. d. Social events, diet, and smoking. 5. A 4-year-old is brought to the emergency department with sudden onset of a temperature of 103°F (39.5°C), sore throat, and refusal to drink. The child will not lie down and prefers to lean forward while sitting up. Which of the following would the nurse do next? a. Give 600 mg acetaminophen (Tylenol) per rectum as ordered. b. Inspect the child’s throat for redness and swelling. c. Have an appropriate-sized tracheostomy tube readily available. d. Obtain a specimen for a throat culture. 6. Assessment of a client taking lithium reveals dry mouth, nausea, thirst, and mild hand tremor. Based on an analysis of these findings, which of the following would the nurse do next? a. Hold the lithium and obtain a stat lithium level to determine therapeutic effectiveness. b. Continue the lithium and immediately notify the physician about the assessment findings. c. Continue the lithium and reassure the client that these temporary side
  • 41. effects will subside. d. Hold the lithium and monitor the client for signs and symptoms of increasing toxicity. 7. A client asks the nurse how long she will have to take her medicine for hypothyroidism. The nurse’s response is based on the knowledge that a. lifelong daily medicine is necessary. b. the medication is expensive, and the dose can be reduced in a few months. c. the medication can be gradually withdrawn in 1 to 2 years. d. the medication can be discontinued after the client’s thyroid-stimulating hormone (TSH) level is normal. 8. Assessment of which of the following clients would lead the nurse to expect the physician to order an adjustment in lithium dosage? a. A client who continues work as a computer programmer. b. A client who attends college classes. c. A client who is now able to care for his or her children. d. A client who is beginning training for a tennis team. 9. A client admitted with a gastric ulcer has been vomiting bright red blood. His hemoglobin is 5.11 g/dL, and his blood pressure is 100/50 mm Hg. The client and the family state that their religious beliefs do not support the use of blood products and refuse blood transfusions as a treatment for the bleeding. The nurse would expect that the next step in the treatment plan would be to a. discontinue all measures. b. notify the hospital attorney. c. attempt to stabilize the client through the use of fluid replacement. d. give enough blood to keep the client from dying. 10. The parents of a child with cystic fibrosis express concern about how the disease was transmitted to their child. The nurse would explain that a. a disease carrier also has the disease. b. two parents who are carriers may produce a child who has the disease. c. a disease carrier and an affected person will never have children with the disease. d. a disease carrier and an affected person will have a child with the disease. 11. A client with angina shows the nurse her nitroglycerin (Nitrostat) that she is carrying in a plastic bag in her pocket. The nurse instructs the client that nitroglycerin should be kept a. in the refrigerator. b. in a cool, moist place.
  • 42. c. in a dark container to shield from light. d. in a plastic bag where it is readily available 12. The nurse caring for client on the telemetry unit is able to determine that the client is in sinus bradycardia by recognizing which characteristics? Select all that apply. a. P wave present. b. Ventricular rate of 50 beats per minute (bpm). c. Atrial rate of 120 bpm. d. PR interval ranging from 0.12 to 0.20. 13. When teaching a client with bipolar disorder, mania, who has started to take valproic acid (Depakene) about possible side effects of this medication, the nurse would include which of the following in the teaching plan? a. Increased urination. b. Slowed thinking. c. Sedation. d. Weight loss. 14. An infant is born with facial abnormalities, growth retardation, mental retardation, and vision abnormalities. These abnormalities are probably caused by maternal a. alcohol consumption b. vitamin B6 deficiency. c. vitamin A deficiency. d. folic acid deficiency. 15. Nonsteroidal anti-inflammatory drugs (NSAIDs) are frequently used in the treatment of musculoskeletal conditions. It is important for the nurse to remind the client to a. take NSAIDs at least three times per day. b. exercise the joints at least 1 hour after taking the medication. c. take antacids 1 hour after taking NSAIDs. d. take NSAIDs with food. 16. The nurse would suspect that the client taking disulfiram (Antabuse) therapy has ingested alcohol when the client exhibits which of the following symptoms? a. Sore throat and muscle aches. b. Nausea and flushing of the face and neck. c. Fever and muscle soreness. d. Bradycardia and vertigo.
  • 43. 17. The nurse holds the gauze pledget against an intramuscular injection site while removing the needle from the muscle. This technique helps to a. seal off the track left by the needle in the tissue. b. speed the spread of the medication in the tissue. c. avoid the discomfort of the needle pulling on the skin. d. prevent organisms from entering the body through the skin puncture. 18. A client whose condition remains stable after a myocardial infarction gradually increases his activity. Which the following conditions should the nurse assess to determine whether the activity is appropriate for the client? a. Edema. b. Cyanosis. c. Dyspnea. d. Weight loss 19. When a client with alcohol dependency begins to talk about not having a problem with alcohol, the nurse would use which of the following approaches? a. Questioning the client about how much alcohol she drinks. b. Confronting the client with the fact that she was intoxicated 2 days ago c. Pointing out how alcohol has gotten her into trouble. d. Listening to what the client states and then asking her how she plans to stay sober. 20. Which of the following correctly describes Medicaid? a. A program designed to assist ill, low-income older adults. b. A federal insurance program for pregnant women. c. A joint federal–state program for low-income persons. d. A program administered by health maintenance organizations. 1. Which individual is at greatest risk for developing hypertension? A) 45 year-old African American attorney B) 60 year-old Asian American shop owner C) 40 year-old Caucasian nurse D) 55 year-old Hispanic teacher 2. A child who ingested 15 maximum strength acetaminophen tablets 45 minutes ago is seen in the emergency department. Which of these orders should the nurse do first?
  • 44. A) Gastric lavage PRN B) Acetylcysteine (mucomyst) for age per pharmacy C) Start an IV Dextrose 5% with 0.33% normal saline to keep vein open D) Activated charcoal per pharmacy 3. Which complication of cardiac catheterization should the nurse monitor for in the initial 24 hours after the procedure? A) angina at rest B) thrombus formation C) dizziness D) falling blood pressure 4. A client is admitted to the emergency room with renal calculi and is complaining of moderate to severe flank pain and nausea. The client’s temperature is 100.8 degrees Fahrenheit. The priority nursing goal for this client is A) Maintain fluid and electrolyte balance B) Control nausea C) Manage pain D) Prevent urinary tract infection 5. What would the nurse expect to see while assessing the growth of children during their school age years? A) Decreasing amounts of body fat and muscle mass B) Little change in body appearance from year to year C) Progressive height increase of 4 inches each year D) Yearly weight gain of about 5.5 pounds per year 6. At a community health fair the blood pressure of a 62 year-old client is 160/96. The client states “My blood pressure is usually much lower.” The nurse should tell the client to A) go get a blood pressure check within the next 48 to 72 hours B) check blood pressure again in 2 months
  • 45. C) see the health care provider immediately D) visit the health care provider within 1 week for a BP check 7. The hospital has sounded the call for a disaster drill on the evening shift. Which of these clients would the nurse put first on the list to be discharged in order to make a room available for a new admission? A) A middle aged client with a history of being ventilator dependent for over 7 years and admitted with bacterial pneumonia five days ago B) A young adult with diabetes mellitus Type 2 for over 10 years and admitted with antibiotic induced diarrhea 24 hours ago C) An elderly client with a history of hypertension, hypercholesterolemia and lupus, and was admitted with Stevens-Johnson syndrome that morning D) An adolescent with a positive HIV test and admitted for acute cellulitus of the lower leg 48 hours ago 8. A client has been newly diagnosed with hypothyroidism and will take levothyroxine (Synthroid) 50 mcg/day by mouth. As part of the teaching plan, the nurse emphasizes that this medication: A) Should be taken in the morning B) May decrease the client’s energy level C) Must be stored in a dark container D) Will decrease the client’s heart rate 9. A 3 year-old child comes to the pediatric clinic after the sudden onset of findings that include irritability, thick muffled voice, croaking on inspiration, hot to touch, sit leaning forward, tongue protruding, drooling and suprasternal retractions. What should the nurse do first? A) Prepare the child for x-ray of upper airways B) Examine the child’s throat C) Collect a sputum specimen D) Notify the healthcare provider of the child’s status 10. In children suspected to have a diagnosis of diabetes, which one of the following complaints would be most likely to prompt parents to take their school age child for evaluation? A) Polyphagia
  • 46. B) Dehydration C) Bed wetting D) Weight loss 11. A client comes to the clinic for treatment of recurrent pelvic inflammatory disease. The nurse recognizes that this condition most frequently follows which type of infection? A) Trichomoniasis B) Chlamydia C) Staphylococcus D) Streptococcus 12. An RN who usually works in a spinal rehabilitation unit is floated to the emergency department. Which of these clients should the charge nurse assign to this RN? A) A middle-aged client who says "I took too many diet pills" and "my heart feels like it is racing out of my chest." B) A young adult who says "I hear songs from heaven. I need money for beer. I quit drinking 2 days ago for my family. Why are my arms and legs jerking?" C) An adolescent who has been on pain medications terminal cancer with an initial assessment finding pupils and a relaxed respiratory rate of 10 D) An elderly client who reports having taken a "large crack hit" 10 minutes prior to walking into the emergency room 13. When teaching a client with coronary artery disease about nutrition, the nurse should emphasize A) Eating 3 balanced meals a day B) Adding complex carbohydrates C) Avoiding very heavy meals D) Limiting sodium to 7 gms per day 14. Which of these findings indicate that a pump to deliver a basal rate of 10 ml per hour plus PRN for pain break through for morphine drip is not working? A) The client complains of discomfort at the IV insertion site
  • 47. B) The client states "I just can’t get relief from my pain." C) The level of drug is 100 ml at 8 AM and is 80 ml at noon D) The level of the drug is 100 ml at 8 AM and is 50 ml at noon 15. The nurse is speaking at a community meeting about personal responsibility for health promotion. A participant asks about chiropractic treatment for illnesses. What should be the focus of the nurse’s response? A) Electrical energy fields B) Spinal column manipulation C) Mind-body balance D) Exercise of joints 16. The nurse is performing a neurological assessment on a client post right CVA. Which finding, if observed by the nurse, would warrant immediate attention? A) Decrease in level of consciousness B) Loss of bladder control C) Altered sensation to stimuli D) Emotional ability 17. A child who has recently been diagnosed with cystic fibrosis is in a pediatric clinic where a nurse is performing an assessment. Which later finding of this disease would the nurse not expect to see at this time? A) Positive sweat test B) Bulky greasy stools C) Moist, productive cough D) Meconium ileus 18. The home health nurse visits a male client to provide wound care and finds the client lethargic and confused. His wife states he fell down the stairs 2 hours ago. The nurse should A) Place a call to the client’s health care provider for instructions B) Send him to the emergency room for evaluation C) Reassure the client’s wife that the symptoms are transient
  • 48. D) Instruct the client’s wife to call the doctor if his symptoms become worse 19. Which of the following should the nurse implement to prepare a client for a KUB (Kidney, Ureter, Bladder) radiograph test? A) Client must be NPO before the examination B) Enema to be administered prior to the examination C) Medicate client with Lasix 20 mg IV 30 minutes prior to the examination D) No special orders are necessary for this examination 20. The nurse is giving discharge teaching to a client 7 days post myocardial infarction. He asks the nurse why he must wait 6 weeks before having sexual intercourse. What is the best response by the nurse to this question? A) "You need to regain your strength before attempting such exertion." B) "When you can climb 2 flights of stairs without problems, it is generally safe.” C) "Have a glass of wine to relax you, then you can try to have sex." D) "If you can maintain an active walking program, you will have less risk." 21. A triage nurse has these 4 clients arrive in the emergency department within 15 minutes. Which client should the triage nurse send back to be seen first? A) A 2 month old infant with a history of rolling off the bed and has bulging fontanels with crying B) A teenager who got a singed beard while camping C) An elderly client with complaints of frequent liquid brown colored stools D) A middle aged client with intermittent pain behind the right scapula 22. While planning care for a toddler, the nurse teaches the parents about the expected developmental changes for this age. Which statement by the mother shows that she understands the child’s developmental needs? A) "I want to protect my child from any falls." B) "I will set limits on exploring the house." C) "I understand the need to use those new skills."
  • 49. D) "I intend to keep control over our child." 23. The nurse is preparing to administer an enteral feeding to a client via a nasogastric feeding tube. The most important action of the nurse is A) Verify correct placement of the tube B) Check that the feeding solution matches the dietary order C) Aspirate abdominal contents to determine the amount of last feeding remaining in stomach D) D) Ensure that feeding solution is at room temperature 24. The nurse is caring for a client with a serum potassium level of 3.5 mEq/L. The client is placed on a cardiac monitor and receives 40 mEq KCL in 1000 ml of 5% dextrose in water IV. Which of the following EKG patterns indicates to the nurse that the infusions should be discontinued? A) Narrowed QRS complex B) Shortened "PR" interval C) Tall peaked T waves D) Prominent "U" waves 25. A nurse prepares to care for a 4 year-old newly admitted for rhabdomyosarcoma. The nurse should alert the staff to pay more attention to the function of which area of the body? A) All striated muscles B) The cerebellum C) The kidneys D) The leg bones 26. The nurse anticipates that for a family who practices Chinese medicine the priority goal would be to A) Achieve harmony B) Maintain a balance of energy C) Respect life D) Restore yin and yang
  • 50. 27. During an assessment of a client with cardiomyopathy, the nurse finds that the systolic blood pressure has decreased from 145 to 110 mm Hg and the heart rate has risen from 72 to 96 beats per minute and the client complains of periodic dizzy spells. The nurse instructs the client to A) Increase fluids that are high in protein B) Restrict fluids C) Force fluids and reassess blood pressure D) D) Limit fluids to non-caffeine beverages 28. A client has a Swan-Ganz catheter in place. The nurse understands that this is intended to measure A) Right heart function B) Left heart function C) Renal tubule function D) Carotid artery function 29. A nurse enters a client’s room to discover that the client has no pulse or respirations. After calling for help, the first action the nurse should take is A) Start a peripheral IV B) Initiate closed-chest massage C) Establish an airway D) Obtain the crash cart 30. A client is receiving digoxin (Lanoxin) 0.25 mg. Daily. The health care provider has written a new order to give metoprolol (Lopressor) 25 mg. B.I.D. In assessing the client prior to administering the medications, which of the following should the nurse report immediately to the health care provider? A) Blood pressure 94/60 B) Heart rate 76 C) Urine output 50 ml/hour D) Respiratory rate 16
  • 51. 31. While assessing a 1 month-old infant, which finding should the nurse report immediately? A) Abdominal respirations B) Irregular breathing rate C) Inspiratory grunt D) Increased heart rate with crying 32. The nurse practicing in a maternity setting recognizes that the post mature fetus is at risk due to A) Excessive fetal weight B) Low blood sugar levels C) Depletion of subcutaneous fat D) Progressive placental insufficiency 33. The nurse is caring for a client who had a total hip replacement 4 days ago. Which assessment requires the nurse’s immediate attention? A) I have bad muscle spasms in my lower leg of the affected extremity. B) "I just can’t ‘catch my breath’ over the past few minutes and I think I am in grave danger." C) "I have to use the bedpan to pass my water at least every 1 to 2 hours." D) "It seems that the pain medication is not working as well today." 34. A client has been taking furosemide (Lasix) for the past week. The nurse recognizes which finding may indicate the client is experiencing a negative side effect from the medication? A) Weight gain of 5 pounds B) Edema of the ankles C) Gastric irritability D) Decreased appetite 35. A client who is pregnant comes to the clinic for a first visit. The nurse gathers data about her obstetric history, which includes 3 year-old twins at
  • 52. home and a miscarriage 10 years ago at 12 weeks gestation. How would the nurse accurately document this information? A) Gravida 4 para 2 B) Gravida 2 para 1 C) Gravida 3 para 1 D) Gravida 3 para 2 36. The nurse is caring for a client with a venous stasis ulcer. Which nursing intervention would be most effective in promoting healing? A) Apply dressing using sterile technique B) Improve the client’s nutrition status C) Initiate limb compression therapy D) Begin proteolytic debridement 37. A nurse is to administer meperidine hydrochloride (Demerol) 100 mg, atropine sulfate (Atropisol) 0.4 mg, and promethizine hydrochloride (Phenergan) 50 mg IM to a pre-operative client. Which action should the nurse take first? A) Raise the side rails on the bed B) Place the call bell within reach C) Instruct the client to remain in bed D) D) Have the client empty bladder 38. Which of these statements best describes the characteristic of an effective reward-feedback system? A) Specific feedback is given as close to the event as possible B) Staff are given feedback in equal amounts over time C) Positive statements are to precede a negative statement D) Performance goals should be higher than what is attainable 39. A client with multiple sclerosis plans to begin an exercise program. In addition to discussing the benefits of regular exercise, the nurse should caution the client to avoid activities which
  • 53. A) Increase the heart rate B) Lead to dehydration C) Are considered aerobic D) May be competitive 40. During the evaluation of the quality of home care for a client with Alzheimer’s disease, the priority for the nurse is to reinforce which statement by a family member? A) At least 2 full meals a day is eaten. B) We go to a group discussion every week at our community center. C) We have safety bars installed in the bathroom and have 24 hour alarms on the doors. D) The medication is not a problem to have it taken 3 times a day. Answer part 2 1. ANSWER D. Symptoms of pregnancy-induced hypertension (PIH), such as hypertension and facial swelling, before 24 weeks’ gestation and a fundal height larger than expected are suggestive of gestational trophoblastic disease or hydatidiform mole. This condition occurs when the trophoblasts develop abnormally. Ultrasound can confirm the condition. Medical management includes evacuation of the mole and follow-up to detect any malignant changes. Painless vaginal bleeding is suggestive of placenta previa. Fetal anemia is often caused by Rh sensitization. Clients with multifetal pregnancies may exhibit larger fundal heights than expected, but they usually do not have a brownish discharge or symptoms of PIH at this stage of gestation. 2. ANSWER A. Saying, “When you interrupt others, they leave the area,” is most helpful because it serves to increase the client’s awareness of how others view him by giving him specific feedback about his behavior. The other statements are punitive and authoritative, possibly threatening to the client, and likely to increase defensiveness, decrease self-worth, and increase feelings of guilt. 3. ANSWER D. The trough serum level should be drawn just before the administration of the next intravenous dose of gentamicin sulfate (Garamycin). 4. ANSWER A. Health-promoting strategies for clients with a history of cardiovascular disease require knowledge in three areas: diet, exercise, and
  • 54. medication. Pain management and management of social activities are not usually features of health promotion activities for these clients. 5. ANSWER C. The child is exhibiting signs and symptoms of possible epiglottitis. As a result the child is at high risk for laryngospasm and airway occlusion. Therefore, the nurse should have a tracheostomy tube and setup readily available should the child experience an airway occlusion. Although acetaminophen is an antipyretic, the dosage of 600 mg to be administered rectally is too high. A typical 4-year-old weighs approximately 40 pounds. The recommended dose is 125 mg. When any type of respiratory illness, and especially epiglottitis, is suspected, putting any object, including a tongue depressor for inspection or a cotton-tipped applicator to obtain a throat culture, in the back of the mouth or throat or having the child open the mouth is inappropriate because doing so may predispose the child to laryngospasm or occlusion of the airway by a swollen epiglottis. 6. ANSWER C. The client is exhibiting the side effects associated with lithium therapy that are temporary. Therefore, the nurse would continue the lithium and explain to the client that he or she is experiencing temporary side effects of lithium that will subside. Common side effects of lithium are nausea, dry mouth, diarrhea, thirst, mild hand tremor, weight gain, bloating, insomnia, and lightheadedness. Immediately notifying the physician about these common side effects is not necessary. 7. ANSWER A. Thyroid replacement is a lifelong maintenance therapy. The medication is usually given as one dose in the morning. It cannot be tapered or discontinued, because the client needs thyroid supplementation to maintain health. The medication cannot be discontinued after the TSH level is normal; the dose will be maintained at the level that normalizes the TSH concentration. 8. ANSWER D. A client who is beginning training for a tennis team would most likely require an adjustment in lithium dosage because excessive sweating can increase the serum lithium level, possibly leading to toxicity. Adjustments in lithium dosage would also be necessary when other medications have been added, when an illness with high fever occurs, and when a new diet begins. 9. ANSWER C. The most appropriate response is to continue all treatments and attempt to stabilize the client using fluid replacement without administering blood or blood products. It is imperative that the health care team respect the client’s religious belief and wishes, even if they are not those of the health care team. Discontinuing all measures is not an option. The health care team should continue to provide the best care possible and does not need to notify the attorney.
  • 55. 10. ANSWER B. Cystic fibrosis is the most common inherited disease in children. It is inherited as an autosomal recessive trait, meaning that the child inherits the defective gene from both parents. The chances are one in four for each of this couple’s pregnancies. 11. ANSWER C. Nitroglycerin in all dosage forms (sublingual, transdermal, or intravenous) should be shielded from light to prevent deterioration. Clients should be instructed to keep the nitroglycerin in the dark container that is supplied by the pharmacy, and it should not be removed or placed in any other container. 12. ANSWER A, B, and D. Sinus bradycardia has the following characteristics: 1) P wave is normal and consistent in shape, occurring in front of every QRS complex; 2) a ventricular and atrial rate less than 60 bpm; and 3) a PR interval that is between 0.12 to 0.20 seconds. An atrial rate of 120 bpm indicates tachycardia. ST segment elevation may indicate a myocardial infarction. 13. ANSWER C. Valproic acid (Depakene) causes sedation as well as nausea, vomiting, and indigestion. Sedation is important because the client needs to be cautioned about driving or operating machinery that could be dangerous while feeling sedated from the medication. Depakene does not cause an increase in urination, slowed thinking, or weight loss. However, some clients may experience weight gain. 14. ANSWER A. These effects and others when seen after birth are known as a cluster of symptoms called fetal alcohol syndrome. Vitamin B6 and vitamin Adeficiency can affect growth and development but not with these specific effects. Folic acid deficiency contributes to neural tube defects. 15. ANSWER D. NSAIDs are irritating to the gastric mucosa and should be taken with food. NSAIDs are usually taken once or twice daily. Joint exercise is not related to the drug administration. Antacids may interfere with the absorption of the drug. 16. ANSWER B. The client who drinks alcohol while taking disulfiram (Antabuse) will experience sweating, flushing of the neck and face, tachycardia, hypotension, a throbbing headache, nausea and vomiting, palpitations, dyspnea, tremor, and/or weakness. 17. ANSWER C. Holding the gauze pledget against an intramuscular injection site while removing the needle from the muscle avoids the discomfort of the needle pulling on the skin. 18. ANSWER C. Physical activity is gradually increased after a myocardial infarction while the client is still hospitalized and through a period of
  • 56. rehabilitation. The client is progressing too rapidly if activity significantly changes respirations, causing dyspnea, chest pain, a rapid heartbeat, or fatigue. When any of these symptoms appears, the client should reduce activity and progress more slowly. Edema suggests a circulatory problem that must be addressed but doesn’t necessarily indicate overexertion. Cyanosis indicates reduced oxygen-carrying capacity of red blood cells and indicates a severe pathology. It is not appropriate to use cyanosis as an indicator for overexertion. Weight loss is indicative of several factors but not overexertion. 19. ANSWER C. When a client talks about not having a problem with alcohol, the nurse needs to point out how alcohol has gotten the client into trouble. Concrete, factual information is helpful in decreasing the client’s denial that alcohol is a problem. The other approaches allow the client to use defense mechanisms, such as rationalization, projection, and minimization, to explain her actions. Therefore, these approaches are not helpful. 20. ANSWER C. Medicaid is state funded, with matching federal funds, and provides medical assistance for low-income persons without health insurance. The program for older adults is Medicare.